Detailed Solutions CSE-PRELIMS 8 • PAPER-1 · 2018-06-07 · 2 | Detailed of Solutions of...

57
BIG LEARNINGS MADE EASY www.nextias.com Detailed Solutions Saket Centre: 316/274, Westend Marg, Saidulajab, Near Saket Metro Station, New Delhi-110030 Old Rajinder Nagar Centre: Ground Floor, 6, Old Rajinder Nagar (Near Salwan School Gate No. 2), New Delhi 110060 Ph: 8800338066 | S CSE-PRELIM 2018 • PAPER-1

Transcript of Detailed Solutions CSE-PRELIMS 8 • PAPER-1 · 2018-06-07 · 2 | Detailed of Solutions of...

Page 1: Detailed Solutions CSE-PRELIMS 8 • PAPER-1 · 2018-06-07 · 2 | Detailed of Solutions of CSE-Prelims:2018 • Paper-1 Analysis of 2018 Prelims: 1. The question paper was difficult

An initiative of Group

B I G L E A R N I N G S M A D E E ASY

www.nextias.com

Detailed Solutions

Saket Centre: 316/274, Westend Marg, Saidulajab, Near Saket Metro Station, New Delhi-110030

Old Rajinder Nagar Centre: Ground Floor, 6, Old Rajinder Nagar (Near Salwan School Gate No. 2), New Delhi 110060

Ph: 8800338066 |

S CSE-PRELIM 2018 • PAPER-1

Page 2: Detailed Solutions CSE-PRELIMS 8 • PAPER-1 · 2018-06-07 · 2 | Detailed of Solutions of CSE-Prelims:2018 • Paper-1 Analysis of 2018 Prelims: 1. The question paper was difficult

2 | Detailed of Solutions of CSE-Prelims:2018 • Paper-1

Analysis of 2018 Prelims:

1. The question paper was difficult as compared to the last year’s Prelims paper.

2. There were many questions where options were set in such a way that it was not easy to solve the question using elimination.

3. There was focus on questions from the Current Affairs and issues in the news. But a considerable number of questions were asked based on events which were in news 2-3 years ago.

4. The questions from conventional topics like Modern Indian History, Science and Technology, Indian Economy etc required in-depth knowledge of the topics.

5. This year, around 22 questions were asked from History section (including Ancient, Medieval, Modern History, Post-Independence and Art & Culture).

6. In last three years UPSC has started asking more questions from Modern India Personalities, their work and organizations.

7. The share of questions based on environment and ecology has reduced in last 2 years. Moreover, in the environment and ecology section questions mainly focused on the environment organizations, treaties and environmental degradation.

8. Flagship schemes of government were asked with their detailed provisions like RTE, PMKVY etc. Moreover, the details asked in these questions are generally not covered in regular newspapers.

Suggestions based on current paper

1. As UPSC continuously changes the pattern of questions asked in the examination, it is recommended that the student should work on his/her conceptual clarity and analytical ability, with respect to both static and current affairs part.

2. In depth study of the topics of current affairs is required, which is not only useful from prelims point of view but also from mains point of view.

3. However, some questions can be solved using common sense and applying knowledge in the examination. The skill to solve objective questions should be developed through regular practice of MCQs.

4. UPSC frequently varies the weightage of the major areas of syllabus, thus it is advised to give equal focus to all areas.

5. Student should also work on his/her temperament and exam management skills. Since UPSC is giving options too close and playing with words (e.g. Arthur Wellesley instead of Richard Wellesley), it is important to read each and every word/statement carefully.

6. CUT-OFF for this year will be lower than last year most likely to be in the range of 97-102 marks. But seeing the unpredictable nature of examination, it is advised that candidates who are getting marks in 90s should continue preparations for the mains examination.

Note:1. Applied Current means inspired by current events or mix of current and traditional.2. Applied traditional means conceptual and application based questions from traditional part.

Page 3: Detailed Solutions CSE-PRELIMS 8 • PAPER-1 · 2018-06-07 · 2 | Detailed of Solutions of CSE-Prelims:2018 • Paper-1 Analysis of 2018 Prelims: 1. The question paper was difficult

Detailed of Solutions of CSE-Prelims:2018 • Paper-1 | 3

Page 4: Detailed Solutions CSE-PRELIMS 8 • PAPER-1 · 2018-06-07 · 2 | Detailed of Solutions of CSE-Prelims:2018 • Paper-1 Analysis of 2018 Prelims: 1. The question paper was difficult

4 | Detailed of Solutions of CSE-Prelims:2018 • Paper-1

Q. No. QUESTION Answer Subject/Topic Nature Topic/A

rea Explanation Source Level

1

Consider the following events :

1. The first democratically elected communistparty government formed in a State in India.2. India’s then largest bank, ‘Imperial Bank ofIndia’, was renamed ‘State Bank of India’.3. Air India was nationalised and became thenational carrier.4. Goa became a part of independent India.

Which of the following is the correct chronologicalsequence of the above events?(a) 4-1-2-3(b) 3-2-1-4(c) 4-2-1-3(d) 3-1-2-4

bPost

Independence

Traditional Factual

Post Indepen

ence

Tata Sons set up Tata Airlines in1932. In 1946, Tata Airlines becamea public company and was renamedAir India. In 1953, air India wasnationalized and became thenational carrier.SBI is the oldest and largest publicsector bank in the Indian bankingsector. The Bank of Calcutta, theBank of Bombay and the Bank ofMadras were merged by an act oflegislature to form the Imperial Bankof India in 1921. In 1955, theImperial Bank of India wasnationalized and renamed as SBI.In 1957, the Communist Party ofIndia (CPI), led by EMSNamboodiripad became one of thefirst democratically electedCommunist governments in theworld, winning elections Kerala.The Portuguese invaded Goa in1510, and ruled it for over fourcenturies. Goa was liberated fromPortuguese rule in 1961.

https://rbidocs.rbi.org.in/rdocs/content/PDFs/90028.pdfLaxmikanth 5th edition page 5.5http://ncert.nic.in/ncerts/l/leps202.pdf,http://www.airindia.in/timeline.htmhttps://www.nseindia.com/content/corporate/eq_SBIN_base.pdf

Difficult

2

Right to Privacy is protected as an intrinsic part ofRight to Life and Personal Liberty. Which of thefollowing in the Constitution of India correctly andappropriately imply the above statement?

(a) Article 14 and the provisions under the 42ndAmendment to the Constitution(b) Article 17 and the Directive Principles of StatePolicy in Part IV(c) Article 21 and the freedoms guaranteed in PartIII(d) Article 24 and the provisions under the 44thAmendment to the Constitution

c Polity and Governance

Current Applied

Fundamental

Rights

In 2017, A nine-judge bench of theSupreme Court ruled that Indiansenjoy a fundamental right to privacy,that it is intrinsic to life and libertyand thus comes under Article 21 ofthe Indian constitution.

Page 175 5th edition Laxmikanthhttps://www.thehindubusinessline.com/news/privacy-is-intrinsic-to-life-and-liberty-rules-apex-court/article9829145.ece

Easy

Page 5: Detailed Solutions CSE-PRELIMS 8 • PAPER-1 · 2018-06-07 · 2 | Detailed of Solutions of CSE-Prelims:2018 • Paper-1 Analysis of 2018 Prelims: 1. The question paper was difficult

Detailed of Solutions of CSE-Prelims:2018 • Paper-1 | 5

3

Consider the following :

1. Areca nut 2. Barley 3. Coffee 4. Finger millet 5. Groundnut 6. Sesamum 7. Turmeric

The Cabinet Committee on Economic Affairs hasannounced the Minimum Support Price for whichof the above?

(a) 1, 2, 3 and 7 only(b) 2, 4, 5 and 6 only(c) 1, 3, 4, 5 and 6 only(d) 1, 2, 3, 4, 5, 6 and 7

bEconomic and Social

Development

Current Applied

Schemes and

Policies

The commodities included in the listdeclared by Cabinet Committee onEconomic Affairs are-

Kharif Crops: Paddy Common,Paddy (F)/Grade'A', Jowar-Hybrid,Jowar-Maldandi, Bajra, Ragi, Maize,Tur, Moong, Urad, groundnut,Sunflower seed, Soyabean black,soyabean yellow, Sesamum,Nigerseed, Medium staple cotton,long staple cotton,

Rabi Crops: Wheat, barley, Gram,Lentil, Mustard, Safflower

Other Commercial Crops: Jute,Sugarcane, Copra (Milling), Copra(Ball)

https://cacp.dacnet.nic.in/ViewContents.aspx?Input=1&PageId=36&KeyId=0http://vikaspedia.in/agriculture/market-information/minimum-support-price

Medium

4

In which one of the following States is PakhuiWildlife Sanctuary located?

(a) Arunachal Pradesh(b) Manipur(c) Meghalaya(d) Nagaland

a Ecology and Environment Pure Current Protecte

d Areas

Pakhui Wildlife Sanctuary (Pakke)lies in the foothills of the EasternHimalaya in the East Kameng Districtof Arunachal Pradesh. It wasdeclared a sanctuary in 1977, andwas earlier part of the KhellongForest Division. It was declared atiger reserve in 2002 based on aproposal in 1999.

http://www.arunachaltourism.com/pakhui-wildlife-sanctuary.phphttp://wiienvis.nic.in/Database/biodiversity_pas_8557.aspx

Easy

Page 6: Detailed Solutions CSE-PRELIMS 8 • PAPER-1 · 2018-06-07 · 2 | Detailed of Solutions of CSE-Prelims:2018 • Paper-1 Analysis of 2018 Prelims: 1. The question paper was difficult

6 | Detailed of Solutions of CSE-Prelims:2018 • Paper-1

5

With reference to India’s satellite launch vehicles,consider the following statements :1. PSLVs launch the satellites useful for Earthresources monitoring whereas GSLVs aredesigned mainly to launch communicationsatellites.2. Satellites launched by PSLV appear to remainpermanently fixed in the same position in the sky,as viewed from a particular location on Earth.3. GSLV Mk III is a four-stage launch vehicle withthe first and third stages using solid rocket motors,and the second and fourth stages using liquidrocket engines.

Which of the statements given above is/arecorrect?(a) 1 only(b) 2 and 3(c) 1 and 2(d) 3 only

a Science and Technology

Current Applied

Technology

Both PSLV (Polar Satellite LaunchVehicle) and GSLV(Geosynchronous Satellite LaunchVehicle) are the satellite-launchvehicles (rockets) developed byISRO. Statement 1 is correct: PSLV isdesigned mainly to deliver the “earth-observation” or “remote-sensing”satellites. The GSLV is designedmainly to deliver the communication-satellites to the highly elliptical(typically 250 x 36000 Km)Geosynchronous Transfer Orbit(GTO).

Statement 2 is not correct: Due totheir geo-synchronous nature, thesatellites launched by GSLV in theseorbits appear to remain permanentlyfixed in the same position in the sky,as viewed from a particular locationon Earth, thus avoiding the need of atracking ground antenna and henceare useful for the communicationapplications.Statement 3 is not correct: GSLVMk III is a three-stage heavy liftlaunch vehicle developed by ISRO.The vehicle has two solid strap-ons,a core liquid booster and a cryogenicupper stage.

https://www.isro.gov.in/launchers, https://www.isro.gov.in/launchers/gslv-mk-iiihttp://www.thehindu.com/sci-tech/science/what-is-the-difference-between-gslv-and-pslv/article6742299.ece

Medium

Page 7: Detailed Solutions CSE-PRELIMS 8 • PAPER-1 · 2018-06-07 · 2 | Detailed of Solutions of CSE-Prelims:2018 • Paper-1 Analysis of 2018 Prelims: 1. The question paper was difficult

Detailed of Solutions of CSE-Prelims:2018 • Paper-1 | 7

6

With reference to the governance of public sectorbanking in India, consider the followingstatements:

1. Capital infusion into public sector banks by theGovernment of India has steadily increased in thelast decade.2. To put the public sector banks in order, themerger of associate banks with the parent StateBank of India has been affected.

Which of the statements given above is/arecorrect?(a) 1 only(b) 2 only(c) Both 1 and 2(d) Neither 1 nor 2

bEconomic and Social

Development

Current Applied

Banking Sector

Government infuses capital into PSUbanks to support credit expansionand to help them tide over lossesresulting from provisions that are tobe made for non-performing assets(NPAs).Statement 1 is not correct: Capital infusion into public sector banks bythe government of India has notsteadily increased in the last decade.There are instances where the therewas a dip in the value of amountgiven by the government for capitalinfusion of public sector banks eg-when we see the trend we can seethat for the year 2009-10 the amountwas 1200 crore, for 2010-11 it was20117.23, and for the year 2011-12the amount was 12000 crore, fromhere we can see that the totalamount increased from 2009-10 to2010-11, then from 2010-11 to 2011-12, the amount again decreased. Statement 2 is correct: To put thepublic sector banks in order, themerger of associate banks with theparent State Bank of India has beenaffected.

https://rbi.org.in/SCRIPTS/PublicationsView.aspx?id=18060,https://www.rbi.org.in/scripts/BS_PressReleaseDisplay.aspx?prid=39884,http://pib.nic.in/newsite/PrintRelease.aspx?relid=158491https://data.gov.in/catalog/capital-infusion-public-sector-banks-government

Difficult

Page 8: Detailed Solutions CSE-PRELIMS 8 • PAPER-1 · 2018-06-07 · 2 | Detailed of Solutions of CSE-Prelims:2018 • Paper-1 Analysis of 2018 Prelims: 1. The question paper was difficult

8 | Detailed of Solutions of CSE-Prelims:2018 • Paper-1

7

Consider the following items:

1. Cereal grains hulled2. Chicken eggs cooked3. Fish processed and canned4. Newspapers containing advertising material

Which of the above items is/are exempted underGST (Goods and Services Tax) ?

(a) 1 only

(b) 2 and 3 only

(c) 1, 2 and 4 only

(d) 1, 2, 3 and 4

cEconomic and Social

Development

Current Applied

Tax Structur

e

Out of the given items, Cereal grainshulled, Chicken eggs cooked andNewspapers containing advertisingmaterial are exempted under theGST tax regime.

http://gstcouncil.gov.in/sites/default/files/NOTIFICATION%20PDF/goods-rates-booklet-03July2017.pdf Difficult

8

Consider the following statements :

1. The definition of “Critical Wildlife Habitat” isincorporated in the Forest Rights Act, 2006.2. For the first time in India, Baigas have beengiven Habitat Rights.3. Union Ministry of Environment, Forest andClimate Change oflicially decides and declaresHabitat Rights for Primitive and Vulnerable TribalGroups in any part of India.

Which of the statements given above is/arecorrect?(a) 1 and 2 only(b) 2 and 3 only(c) 3 only(d) 1, 2 and 3

a Ecology and Environment

Traditional Conceptual

Governance

Statement 1 is correct: The term"Critical Wildlife Habitat" isincorporated in the Forests RightsAct, 2006.Statement 2 is correct: Living in thedense sal forests of Maikal Hills,where streams intersect theirhomesteads, the Baigas in Dindoridistrict of Madhya Pradesh havegained the right to their habitat. Thisis for the first time habitat rights havebeen given under the Forest RightsAct of 2006. For the Baigas, it is aPyrrhic victory that has come after astruggle for more than a century.Statement 3 is not correct: Ministryof Tribal Affairs officially decides anddeclares Habitat Rights for Primitiveand Vulnerable tribal Groups in anypart on India.

http://pib.nic.in/newsite/PrintRelease.aspx?relid=69806,https://tribal.nic.in/pvtg.aspx

Medium

Page 9: Detailed Solutions CSE-PRELIMS 8 • PAPER-1 · 2018-06-07 · 2 | Detailed of Solutions of CSE-Prelims:2018 • Paper-1 Analysis of 2018 Prelims: 1. The question paper was difficult

Detailed of Solutions of CSE-Prelims:2018 • Paper-1 | 9

9

Consider the following :

1. Birds 2. Dust blowing 3. Rain 4. Wind blowing

Which of the above spread plant diseases?(a) 1 and 3 only(b) 3 and 4 only(c) 1, 2 and 4 only(d) 1, 2, 3 and 4

d Ecology and Environment

Traditional Conceptual Ecology

Most disease-causing viruses arecarried and transmitted naturally byinsects and mites, which are calledvectors of the virus. Slugs, snails,birds, rabbits, and dogs also transmita few viruses, but this is notcommon. Diseases that attack theleaves of a plant are primarily spreadby wind, but they can also move tonearby plants by taking a ride onsplashing water droplets from rain orirrigation. Dust blowing also playsrole in spreading plant diseases.

https://www.britannica.com/science/plant-disease/Transmissionhttps://www.sciencefriday.com/educational-resources/how-do-diseases-spread-between-plants/

Easy

10

With reference to organic farming in India,consider the following statements:

1.‘The National ‘Programme for OrganicProduction’ (NPOP) is operated under theguidelines and‘ directions of the Union Ministry ofRural Development.2.‘The Agricultural and Processed Food ProductExport Development Authority‘ (APEDA) functionsas the Secretariat for the implementation ofNPOP.3. Sikkim has become India’s first fully organicState.

Which of the statements given above is/arecorrect?

(a) 1 and 2 only(b) 2 and 3 only(c) 3 only(d) 1, 2 and 3

bEconomic and Social

DevelopmentPure Current

Schemes and

Policies

Statement 1 is not correct: Ministryof Commerce has implemented theNational Programme for OrganicProduction (NPOP) since 2001. Statement 2 is correct: APEDAshall function as the Secretariat forthe implementation of the NPOP. Statement 3 is correct: Sikkimbecame India's first fully organicstate in July 2016.

http://apeda.gov.in/apedawebsite/organic/index.htm, http://pib.nic.in/PressReleseDetail.aspx?PRID=1508118,http://pib.nic.in/newsite/PrintRelease.aspx?relid=155000http://apeda.gov.in/apedawebsite/organic/organic_contents/Chapter_2.pdfhttp://www.thehindu.com/news/national/Sikkim-becomes-India%E2%80%99s-first-organic-state/article13999445.ece

Medium

Page 10: Detailed Solutions CSE-PRELIMS 8 • PAPER-1 · 2018-06-07 · 2 | Detailed of Solutions of CSE-Prelims:2018 • Paper-1 Analysis of 2018 Prelims: 1. The question paper was difficult

10 | Detailed of Solutions of CSE-Prelims:2018 • Paper-1

11

Regarding Money Bill, which of the followingstatements is not correct?

(a) A bill shall be deemed to be a Money Bill if itcontains only provisions relating to imposition,abolition, remission, alteration or regulation of anytax.(b) A Money Bill has provisions for the custody ofthe Consolidated Fund of India or the ContingencyFund of India.(c) A Money Bill is concerned with theappropriation of money out of the ContingencyFund of India.(d) A Money Bill deals with the regulation ofborrowing of money or giving of any guarantee bythe Government of India.

c Polity and Governance

Traditional Factual

Parliament

Article 110 of the Constitution dealswith the definition of money bills. Itstates that a bill is deemed to be amoney bill if it contains ‘only’provisions dealing with all or any ofthe following matters: 1. Theimposition, abolition, remission,alteration or regulation of any tax(Option a is correct); 2. Theregulation of the borrowing of moneyby the Union government (Option dis correct); 3. The custody of theConsolidated Fund of India or thecontingency fund of India, thepayment of moneys into or thewithdrawal of money from any suchfund (Option b is correct); 4. Theappropriation of money out of theConsolidated Fund of India; 5.Declaration of any expenditurecharged on the Consolidated Fund ofIndia or increasing the amount of anysuch expenditure; 6. The receipt ofmoney on account of theConsolidated Fund of India or thepublic account of India or thecustody or issue of such money, orthe audit of the accounts of theUnion or of a state; or 7. Any matterincidental to any of the mattersspecified above

Laxmikanth Fifth edition, page no. 22.19 Easy

Page 11: Detailed Solutions CSE-PRELIMS 8 • PAPER-1 · 2018-06-07 · 2 | Detailed of Solutions of CSE-Prelims:2018 • Paper-1 Analysis of 2018 Prelims: 1. The question paper was difficult

Detailed of Solutions of CSE-Prelims:2018 • Paper-1 | 11

12

With reference to the election of the President ofIndia, consider the following statements:

1. The value of the vote of each MLA varies fromState to State.2. The value of the vote of MPs of the Loksabha ismore than the value of the vote of MPs of theRajyasabha.Which of the following statements given aboveis/are correct?

(a) 1 only(b) 2 only(c) Both 1 and 2(d)Neither 1 nor 2

a Polity and Governance

Traditional Conceptual

Parliament and State

Legislature

Statement 1 is correct: The formulafor determining the number of votesheld by an MLA is:Statement 2 is not correct: Thevalue of an MP's vote is calculatedby dividing the total value of allMLAs' votes by the number of MPs.The formula for determining thenumber of votes held by an MP is:

Laxmikanth Fifth edition, page no. 17.3 Easy

13

In the Indian context, what is the implication ofratifying the ‘Additional Protocol’ with the‘International Atomic Energy Agency (IAEA)’?

(a) The civilian nuclear reactors come under IAEAsafeguards.(b)The military nuclear installations come underthe inspection of IAEA(c)The country will have the privilege to buyuranium from the Nuclear Suppliers Group (NSG).(d) The country automatically becomes a memberof the NSG.

a International Relations

Current Applied

Institutions and Agencie

s

An Additional Protocol (AP) to theSafeguards Agreement between theGovernment of India and the IAEAfor the Application of Safeguards toCivilian Nuclear Facilities enteredinto force on 25 July 2014.

https://www.iaea.org/newscenter/news/indias-additional-protocol-enters-force

Medium

14

Consider the following countries :

1. Australia2. Canada3. China4. India5. Japan6. USA

Which of the above are among the ‘free-tradepartners’ of ASEAN ?(a) 1, 2, 4 and 5(b) 3, 4, 5 and 6(c) 1, 3, 4 and 5(d) 2, 3, 4 and 6

c International Relations

Traditional Conceptual

International

Organisations/Partnersh

ips

The Association of Southeast AsianNations (ASEAN) has free tradeagreements with six partners namelyPeople’s Republic of China (ACFTA),Republic of Korea (AKFTA), Japan(AJCEP), India (AIFTA) as well asAustralia and New Zealand(AANZFTA).

http://asean.org/?static_post=rcep-regional-comprehensive-economic-partnership

Easy

Total population of the state or union territoryValue of an MLA vote =

Total number of elected numbers of the Legislative Assembly ×100

Total sum of vote value of elected members of all the Legislative AssembliesValue of an MLA vote =

The sum elected members of both the houses of Parliament

Page 12: Detailed Solutions CSE-PRELIMS 8 • PAPER-1 · 2018-06-07 · 2 | Detailed of Solutions of CSE-Prelims:2018 • Paper-1 Analysis of 2018 Prelims: 1. The question paper was difficult

12 | Detailed of Solutions of CSE-Prelims:2018 • Paper-1

15

With reference to the ‘Global Alliance for Climate-Smart Agriculture (GACSA)’, which of the followingstatements is/are correct ?

1. GACSA is an outcome of the Climate Summitheld in Paris in 2015.2. Membership of GACSA does not create anybinding obligations.3. India was instrumental in the creation ofGACSA.

Select the correct answer using the code givenbelow :(a) 1 and 3 only(b) 2 only(c) 2 and 3 only(d) 1, 2 and 3

b Ecology and Environment

Current Applied

Institutions and Agencie

s

Statement 1 is not correct: TheGlobal Alliance for Climate SmartAgriculture (GACSA) was initiatedand established in 2014, in UNClimate Summit 2014.

Statement 2 is correct: Membership of GACSA does not create anybinding obligations.

Statement 3 is not correct: India asa country is not a member ofGACSA.

http://www.fao.org/gacsa/about/en/https://www.norad.no/en/front/thematic-areas/climate-change-and-environment/food-security/right-to-food/https://www.iatp.org/blog/201509/what%E2%80%99s-wrong-with-%E2%80%9Cclimate-smart%E2%80%9D-agriculture

Medium

16

Which of the following is/are the aim/aims of“Digital India” Plan of the Government of India ?

1. Formation of India’s own Internet companieslike China did.2. Establish a policy framework to encourageoverseas multinational corporations that collectBig Data to build their large data centres within ournational geographical boundaries.3. Connect many of our villages to the Internet andbring Wi-Fi to many of our schools, public placesand major tourist centres.

Select the correct answer using the code givenbelow :(a) 1 and 2 only(b) 3 only(c) 2 and 3 only(d) 1, 2 and 3

bEconomic and Social

Development

Current Applied

Schemes and

Policies

Digital India aims to provide themuch needed thrust to the ninepillars of growth areas, namelyBroadband Highways, UniversalAccess to Mobile Connectivity,Public Internet Access Programme, e-Governance: Reforming Governmentthrough Technology, e-Kranti -Electronic Delivery of Services,Information for All, ElectronicsManufacturing, IT for Jobs and EarlyHarvest Programmes. Each of theseareas is a complex programme initself and cuts across multipleMinistries and Departments.

http://vikaspedia.in/e-governance/digital-india/nine-pillars-of-digital-indiahttp://www.digitalindia.gov.in/content/programme-pillars

Easy

Page 13: Detailed Solutions CSE-PRELIMS 8 • PAPER-1 · 2018-06-07 · 2 | Detailed of Solutions of CSE-Prelims:2018 • Paper-1 Analysis of 2018 Prelims: 1. The question paper was difficult

Detailed of Solutions of CSE-Prelims:2018 • Paper-1 | 13

17

Consider the following pairs :

Towns sometimes mentioned in news Country1. AleppoSyria2. KirkukYemen3. MosulPalestine4. Mazar-i-sharifAfghanistan

Which of the pairs given above are correctlymatched?(a) 1 and 2(b) 1 and 4(c) 2 and 3(d) 3 and 4

b Geography Pure Current Mapping

Aleppo is located in Syria, Kirkuk and Mosul are in Iraq and Mazar-i-sharifis in Afghanistan.

http://www.bbc.com/news/world-middle-east-38132163http://www.thehindu.com/todays-paper/tp-international/mazar-i-sharif-afghanistan-april-7-2018-afp/article23469762.ecehttp://www.thehindu.com/news/international/iraqi-forces-complete-takeover-of-kirkuk-province-after-clashing-with-kurds/article19888096.ecehttps://www.aljazeera.com/indepth/inpictures/mosul-body-collectors-180530113119652.html

Easy

18

In the Federation established by The Governmentof India Act of 1935, residuary powers were givento the

(a)Federal Legislature(b) Governor General(c)Provincial Legislature(d) Provincial Governors

bModern Indian History

Traditional Factual

Making of

Constitution

In the federation established by theGovernment of India Act 1935,residuary powers were given to theGovernor General to decide in hissole discretion as to under which lista particular subject fell.

Laxmikanth Fifth edition, page no. 1.7BL Grover Page 404

Easy

Page 14: Detailed Solutions CSE-PRELIMS 8 • PAPER-1 · 2018-06-07 · 2 | Detailed of Solutions of CSE-Prelims:2018 • Paper-1 Analysis of 2018 Prelims: 1. The question paper was difficult

14 | Detailed of Solutions of CSE-Prelims:2018 • Paper-1

19

Consider the following statements :

1. The Speaker of the Legislative Assembly shallvacate his/her office if he/she ceases to be amember of the Assembly.2. Whenever the Legislative Assembly isdissolved, the Speaker shall vacate his/her officeimmediately.

Which of the statements given above is/arecorrect ?(a) 1 only(b) 2 only(c) Both 1 and 2(d) Neither 1 nor 2

a Polity and Governance

Traditional Factual

Parliament

Statement 1 is correct: TheSpeaker remains in office during thelife of the assembly. However, hevacates his office earlier in any of thefollowing three cases: 1. if he ceasesto be a member of the assembly; 2. ifhe resigns by writing to the deputyspeaker; and 3. if he is removed by aresolution passed by a majority of allthe then members of the assembly.Such a resolution can be moved onlyafter giving 14 days advance notice.Statement 2 is not correct:Whenever the Lok Sabha isdissolved, the Speaker does notvacate his office and continues tillthe newly elected Lok Sabha meets.

Laxmikanth Fifth edition, page no 22.7 Easy

20

Which one of the following reflects the mostappropriate relationship between law and liberty ?

(a) If there are more laws, there is less liberty.(b) If there are no laws, there is no liberty.(c) If there is liberty laws have to be made by thepeople.(d) If laws are changed too often, liberty is indanger.

a Polity and Governance

Traditional Conceptual

Political Theory

Freedom is said to exist whenexternal constraints on the individualare absent. In terms of this definitionan individual could be consideredfree if he/she is not subject toexternal controls or coercion and isable to make independent decisionsand act in an autonomous way.

http://www.ncert.nic.in/ncerts/l/keps102.pdf Difficult

Page 15: Detailed Solutions CSE-PRELIMS 8 • PAPER-1 · 2018-06-07 · 2 | Detailed of Solutions of CSE-Prelims:2018 • Paper-1 Analysis of 2018 Prelims: 1. The question paper was difficult

Detailed of Solutions of CSE-Prelims:2018 • Paper-1 | 15

21

Consider the following statements :

1. No criminal proceedings shall be institutedagainst the Governor of a State in any court duringhis term of office.2. The emoluments and allowances of theGovernor of a State shall not be diminished duringhis term of office.

Which of the statements given above is/arecorrect?(a) 1 only(b) 2 only(c) Both 1 and 2(d) Neither 1 nor 2

c Polity and Governance

Traditional Factual

Governor

Statement 1 is correct: No criminalproceedings whatsoever shall beinstituted or continued against thePresident, or theGovernor of a State, in any courtduring his term ofoffice.Statement 2 is correct: Theemoluments and allowances of theGovernorshall not be diminished during histerm of office.

Bare Act Article 361: http://www.legislative.gov.in/sites/default/files/coi-4March2016.pdfBare Act Article 158

Easy

22

The well-known painting “Bani Thani” belongs tothe

(a) Bundi school(b) Jaipur school(c) Kangra school(d) Kishangarh school.

d Art and Culture

Traditional Factual

Paintings

Bani Thani Paintings were in newsrecently because there will be adisplay of famous Bani-Thanipaintings of Kishangarh style in thegalleries and halls of the newly builtKishangarh airport.

The painting of Nihalchand ofKishangarh of Rajasthani School hasa special place in the treasure ofIndian miniature paintings. It is oftencompared to Monalisa supposed toepitomise the ideal of womanhood.

https://timesofindia.indiatimes.com/city/jaipur/bani-thani-paintings-to-welcome-passengers-at-kishangarh-airport/articleshow/58451262.cms

Medium

23

What is “Terminal High Altitude Area Defense(THAAD)”, sometimes seen in the news ?

(a) An Israeli radar system(b) India’s indigenous anti-missile programme(c)An American anti-missile system(d) A defence collaboration between Japan andSouth Korea

c Science and Technology Pure Current

Defence Technol

ogy

THAAD is an addition to the UnitedStates’ anti-ballisticmissile/interceptor toolkit. It enteredproduction in 2008 and is primarilytasked with taking out threateningballistic missiles in what’s known astheir “terminal” phase (the ‘T’ in theacronym).

https://thediplomat.com/2016/02/what-is-thaad-what-does-it-do-and-why-is-china-mad-about-it/

Easy

Page 16: Detailed Solutions CSE-PRELIMS 8 • PAPER-1 · 2018-06-07 · 2 | Detailed of Solutions of CSE-Prelims:2018 • Paper-1 Analysis of 2018 Prelims: 1. The question paper was difficult

16 | Detailed of Solutions of CSE-Prelims:2018 • Paper-1

24

With reference to cultural history of India, considerthe following statements :

1. Most of the Tyagaraja Kritis are devotionalsongs in praise of Lord Krishna.2. Tyagaraja created several new ragas.3. Annamacharya and Tyagaraja arecontemporaries.4. Annamacharya kirtanas are devotional songs inpraise of Lord Venkateshwara.

Which of the statements given above are correct?(a)1 and 3 only(b) 2 and 4 only(c)1, 2 and 3(d) 2, 3 and 4

b Art and Culture

Current Applied

Personalities/Cu

lture

Statement 1 is not correct: Most ofTyagaraja’s songs were in praise ofLord Rama.

Statement 2 is correct: Tyagarajacreated several new ragas.

statement 3 is not correct: :SriTalapakka Annamacharya ofTalapakka village in Cuddapahdistrict was a saintcomposer ofthe15th century, while Tyagaraja wasborn in 1767, and recently his 250thbirth anniversary was celebrated.Statement 4 is correct:Annamacharya kirtanas aredevotional songs in praise of LordVenkateshwara.

https://www.britannica.com/biography/Tyagaraja

https://books.google.co.in/books?isbn=818495445X

Difficult

25

Which of the following are regarded as the mainfeatures of the “Rule of Law” ?

1. Limitation of powers2. Equality before law3. People’s responsibility to the Government4. Liberty and civil rights

Select the correct answer using the code givenbelow :(a) 1 and 3 only(b) 2 and 4 only(c) 1, 2 and 4 only(d) 1, 2, 3 and 4

c Polity and Governance

Traditional Conceptual

Governance

Rule of law includes limitation ofpower, equality before law and libertyand civil rights. What the rule of lawmeans is that all laws apply equallyto all citizens of the country and noone can be above the law. Neither agovernment official, nor a wealthyperson nor even the President of thecountry is above the law.

http://ncert.nic.in/ncerts/l/iess406.pdfhttp://ncert.nic.in/ncerts/l/hess304.pdfhttp://www.ncert.nic.in/ncerts/l/keps102.pdf

Medium

Page 17: Detailed Solutions CSE-PRELIMS 8 • PAPER-1 · 2018-06-07 · 2 | Detailed of Solutions of CSE-Prelims:2018 • Paper-1 Analysis of 2018 Prelims: 1. The question paper was difficult

Detailed of Solutions of CSE-Prelims:2018 • Paper-1 | 17

26

Which of the following led to the introduction ofEnglish Education in India?

1. Charter Act of 18132. General Committee of Public Instruction, 18233. Orientalist and Anglicist Controversy

Select the correct answer using the code givenbelow :(a) 1 and 2 only(b) 2 only(c) 1 and 3 only(d) 1, 2 and3

dModern Indian History

Traditional Factual

Education

All the three given laws/eventsfacilitated introduction of Englisheducation in India.Under Charter Act of 1813, officialmoney of rupees one lac was allottedto expand the education of theIndians, which compelled East IndiaCompany to accept responsibility forthe education of the Indian people.As a result, from 1813 to 1857, thecompany opened many schools andcolleges under their control whichlaid the foundation of the Englishsystem of education in India.General Committee of PublicInstruction, 1823 under chairmanshipof Lord Macauley took a decisivestand against the native culture andlearning, native knowledge andlanguages like Sanskrit, Arabic andPersian and asserted on theeducation of Western Sciencethrough English language.All of those groups who may becalled ‘Anglicists’, in generalbelieved thatIndians were in a backward stageand Western education giventhrough English language alone wasthe remedy, which helped in spreadof English Education in India.

http://www.kkhsou.in/main/education/charter_act.htmlhttp://www.nios.ac.in/media/documents/dled/Block1_501.pdf

Difficult

27

Which one of the following is an ‘artificial lake‘?

(a) Kodaikanal (Tamil Nadu)(b) Kolleru (Andhra Pradesh)(c) Nainital (Uttarakhand)(d) Renuka (Himachal Pradesh)

a Geography Traditional Factual

Water Bodies

Kodaikanal Lake was created in1863 by Sir Hendry Levinge. He waspreviously the Collector of Madurai,who retired and settled inKodaikanal.

http://www.environment.tn.nic.in/doc/Kodaikanal%20Lak1-22.07.09.doc

Medium

Page 18: Detailed Solutions CSE-PRELIMS 8 • PAPER-1 · 2018-06-07 · 2 | Detailed of Solutions of CSE-Prelims:2018 • Paper-1 Analysis of 2018 Prelims: 1. The question paper was difficult

18 | Detailed of Solutions of CSE-Prelims:2018 • Paper-1

28

With reference to Pradhan Mantri Kaushal VikasYojana, consider the following statements :

1. It is the flagship scheme of the Ministry ofLabour and Employment.2. It, among other things, will also impart trainingin soft skills, entrepreneurship, financial and digitalliteracy.3. It aims to align the competencies of theunregulated workforce of the country to theNational Skill Qualification Framework.Which of the statements given above is/arecorrect?

(a) 1 and 3 only(b) 2 only(c) 2 and 3 only(d) 1, 2 and 3

cEconomic and Social

DevelopmentPure Current

Schemes and

Policies

Statement 1 is not correct:Pradhan Mantri Kaushal VikasYojana (PMKVY) is the flagshipscheme of the Ministry of SkillDevelopment & Entrepreneurship(MSDE).Statement 2 is correct: Apart fromproviding training according to theNational Skills QualificationFramework (NSQF), TCs shall alsoimpart training in Soft Skills,Entrepreneurship, Financial andDigital Literacy. Statement 3 is correct: A keycomponent of scheme, Recognitionof Prior Learing (RPL) aims to alignthe competencies of the unregulatedworkforce of the country to theNSQF.

http://www.skilldevelopment.gov.in/pmkvy.html Easy

29

In 1920, which of the following changed its nameto “Swarajya Sabha” ?

(a) All India Home Rule League(b) Hindu Mahasabha(c) South Indian Liberal Federation(d) The Servants of India Society

aModern Indian History

Traditional Factual

Freedom

Struggle

Gandhi secured the presidency ofthe Home Rule League when he wasinvited to chair a meeting in Bombayon 3rd October 1920. There hechanged the name of the associationto Swarajya Sabha, not 'Home Rule'anymore, but an association forindependence.

Gandhi's Passion: The Life and Legacy of Mahatma GandhiBy Stanley Wolper Pg 110Gandhi and Nationalism: The Path to Indian IndependenceBy Simone Panter-Brick pg 32

Difficult

Page 19: Detailed Solutions CSE-PRELIMS 8 • PAPER-1 · 2018-06-07 · 2 | Detailed of Solutions of CSE-Prelims:2018 • Paper-1 Analysis of 2018 Prelims: 1. The question paper was difficult

Detailed of Solutions of CSE-Prelims:2018 • Paper-1 | 19

30

Which among the following events happenedearliest?

(a) Swami Dayanand established Arya Samaj.(b) Dinabandhu Mitra wrote Neeldarpan(c) Bankim Chandra Chattopadhyay wroteAnandmath.(d) Satyendranath Tagore became the first Indianto succeed in the Indian Civil ServicesExamination.

bModern Indian History

Traditional Factual

Personalities of Freedo

m Struggle

Swami Dayanand Saraswatiestablished Arya Samaj in 1875.

In 1860, Dinabandhu Mitra, aninspector of the post officedepartment, published a play fromDhaka titled Neel Darpan.

Bankim Chandra Chattopadhyaywrote Anandmath in 1882.

Satyendranath Tagore became thefirst Indian to succeed in the IndianCivil Services Examination in 1863.

BL Grover Page no. 489, 278

http://www.open.ac.uk/researchprojects/makingbritain/content/satyendranath-tagore

Difficult

Page 20: Detailed Solutions CSE-PRELIMS 8 • PAPER-1 · 2018-06-07 · 2 | Detailed of Solutions of CSE-Prelims:2018 • Paper-1 Analysis of 2018 Prelims: 1. The question paper was difficult

20 | Detailed of Solutions of CSE-Prelims:2018 • Paper-1

31

Which of the following is/are the possibleconsequence/s of heavy sand mining in riverbeds?

1. Decreased salinity in the river2. Pollution of groundwater3. Lowering of the water-table

Select the correct answer using the code givenbelow :(a) 1 only(b) 2 and 3 only(c) 1 and 3 only(d) 1, 2 and 3

b Ecology and Environment

Traditional Conceptual

Water Pollutio

n

In NCR, the illegal sand miningmostly occurs along the Yamunariverbed in the cities of Delhi, Noida,Haryana and the Aravalli ranges.This indiscriminate sand mining inthe river beds of Yamuna & Ghaggarhas threatened the biodiversity,destroyed riverine vegetation,caused erosion, polluted watersources etc. Over the years,extensive damage has been causedto the ecosystem of these rivers, hasweakenedthe river beds, spelleddisaster for the conservation ofmany bird species, increasedsalinity of water in the rivers etc.Statement(1) is not correct: Heavy sand mining in riverbed would lead toincreased salinity in the river.Depletion of sand in thestreambed causes the deepeningof rivers and estuaries, and theenlargement of river mouths andcoastal inlets. It leads to saline-waterintrusion. Statement(2) is correct: Sand mining would lead to pollution ofgroundwater.Statement(3) is correct: Heavy sand mining would lead to loweringof water table.

http://www.nluassam.ac.in/data3/lexterraissue23/Article%202.pdfhttp://envfor.nic.in/sites/default/files/Final%20Sustainable%20Sand%20Mining%20Management%20Guidelines%202016.pdfhttp://shodhganga.inflibnet.ac.in/bitstream/10603/8987/12/12_chapter%208.pdfhttp://www.nluassam.ac.in/data3/lexterraissue23/Article%202.pdf

Medium

Page 21: Detailed Solutions CSE-PRELIMS 8 • PAPER-1 · 2018-06-07 · 2 | Detailed of Solutions of CSE-Prelims:2018 • Paper-1 Analysis of 2018 Prelims: 1. The question paper was difficult

Detailed of Solutions of CSE-Prelims:2018 • Paper-1 | 21

32

With reference to agricultural soils, consider thefollowing statements :

1. A high content of organic matter in soildrastically reduces its water holding capacity.2. Soil does not play any role in the sulphur cycle.3. Irrigation over a period of time can contribute tothe salinization of some agricultural lands.

Which of the statements given above is/arecorrect ?(a) 1 and 2 only(b) 3 only(c) 1 and 3 only(d) 1, 2 and 3

b Ecology and Environment

Traditional Conceptual

Soil Pollutio

n

Statement 1 is not correct: Plantresidues that cover the soil surfaceprotect the soil from sealing andcrusting by raindrop impact, therebyenhancing rainwater infiltration andreducing runoff. Increased organicmatter contributes indirectly to soilporosity (via increased soil faunalactivity).Statement 2 is not correct: Sulfur cycle is circulation of sulfur in variousforms through nature. Sulfur occursin all living matter as a component ofcertain amino acids. It is abundant inthe soil in proteins and, through aseries of microbial transformations,ends up as sulfates usable by plants.

Statement 3 is correct: Secondary salinization occurs when significantamounts of water areprovided by irrigation, with noadequate provision of drainage fortheleaching and removal of salts,resulting in the soils becoming saltyandunproductive.

Statement 1: http://www.fao.org/docrep/009/a0100e/a0100e08.htmStatement 2: https://www.sciencedirect.com/topics/agricultural-and-biological-sciences/sulfur-cyclestatement 3: http://www.fao.org/tempref/agl/agll/docs/salinity_brochure_eng.pdf

Medium

Page 22: Detailed Solutions CSE-PRELIMS 8 • PAPER-1 · 2018-06-07 · 2 | Detailed of Solutions of CSE-Prelims:2018 • Paper-1 Analysis of 2018 Prelims: 1. The question paper was difficult

22 | Detailed of Solutions of CSE-Prelims:2018 • Paper-1

33

The Partnership for Action on Green Economy(PAGE), a UN mechanim to assist countriestransition towards greener and more inclusiveeconomies, emerged at

(a) The Earth Summit on SustainableDevelopment 2002, Johannesburg(b) The United Nations Conference on SustainableDevelopment 2012, Rio de Janeiro(c) The United Nations Framework Convention onClimate Change 2015, Paris(d) The World Sustainable Development Summit2016, New Delhi

b Ecology and Environment Pure Current

International

Institutions/Partnership

s

The Partnership for Action on GreenEconomy (PAGE) was launched in2013 as a response to the call atRio+20 to support those countrieswishing to embark on greener andmore inclusive growth trajectories.

PAGE seeks to put sustainability atthe heart of economic policies andpractices to advance the 2030Agenda for SustainableDevelopment and supports nationsand regions in reframing economicpolicies and practices aroundsustainability to foster economicgrowth, create income and jobs,reduce poverty and inequality, andstrengthen the ecologicalfoundations of their economies.

http://www.un-page.org/about/who-are-we

Medium

Page 23: Detailed Solutions CSE-PRELIMS 8 • PAPER-1 · 2018-06-07 · 2 | Detailed of Solutions of CSE-Prelims:2018 • Paper-1 Analysis of 2018 Prelims: 1. The question paper was difficult

Detailed of Solutions of CSE-Prelims:2018 • Paper-1 | 23

34

“3D printing” has applications in which of thefollowing?

1. Preparation of confectionery items2. Manufacture of bionic ears3. Automotive industry4. Reconstructive surgeries5. Data processing technologies

Select the correct answer using the code givenbelow :(a) 1, 3 and 4 only(b) 2, 3 and 5 only(c) 1 and 4 only(d) 1, 2, 3, 4 and 5

d Science and Technology

Current Applied

Information

Technology

3D printing technologies can be usedto design and build everything fromautomobile parts to medical devicesusing additive manufacturingtechnology.

Because additive manufacturingtypically creates products layer bylayer, manufacturers will also havethe opportunity, and may feel theneed, to monitor production inentirely new ways. Closelymonitoring products as they areformed could yield terabytes of data.CIOs will need to determine whatdata is useful and should bemaintained, and what should bediscarded (and when). The ability tomaintain and analyze valuable datamay become increasingly importantas 3D printing technologies aredeployed in the production of engineparts, medical devices, and otheritems upon which human safetydepends.

https://www.forbes.com/sites/forbestechcouncil/2018/02/06/11-tech-pros-share-their-favorite-applications-of-3d-printing-technology/2/#39ee704f5991

http://deloitte.wsj.com/cio/2013/06/25/3d-printing-data-data-everywhere/

Medium

Page 24: Detailed Solutions CSE-PRELIMS 8 • PAPER-1 · 2018-06-07 · 2 | Detailed of Solutions of CSE-Prelims:2018 • Paper-1 Analysis of 2018 Prelims: 1. The question paper was difficult

24 | Detailed of Solutions of CSE-Prelims:2018 • Paper-1

35

Consider the following statements:

1. The Barren Island volcano is an active volcanolocated in the Indian territory.2. Barren Island lies about 140 km east of GreatNicobar.3. The last time the Barren Island volcano eruptedwas in 1991 and it has remained inactive sincethen.

Which of the statements given above is/arecorrect ?(a) 1 only(b) 2 and 3(c) 3 only(d) 1 and 3

a Geography Current Applied

Indian Geogra

phy

Statement 1 is correct: The BarrenIsland volcano is an active volcanolocated in the Indian territory.Statement 2 is not correct: TheBarren Island, about 140 km fromPort Blair, is a tourist destination andsurrounded by waters ideal for scubadiving and is home to a wide varietyof aquatic life. Port Blair lies in SouthAndaman. Barren Island does not lieto the east of Great Nicobar. It liesapproximately in between MiddleAndaman and South Andaman.Statement 3 is not correct: Barrenisland erupted recently in 2017.Thus, option a is correct.

http://www.thehindu.com/news/national/the-barren-islandvolcano-erupts-again/article17369862.ece

Medium

36

Why is a plant called Prosopis juliflora oftenmentioned in news?

(a) Its extract is widely used in cosmetics.(b) It tends to reduce the biodiversity in the area inwhich it grows.(c) Its extract is used in the synthesis ofpesticides.(d) None of the above

b Ecology and Environment

Current Applied Ecology

Prosopis juliflora, commonly knownas Vilayati Keekar or Seemaikaruvelam is an invasive species. Ittends to reduce the biodiversity inthe area in which it grows.

http://indianexpress.com/article/explained/delhi-government-vilayati-kikar-removal-order-environmental-impact-5194797/

Difficult

Page 25: Detailed Solutions CSE-PRELIMS 8 • PAPER-1 · 2018-06-07 · 2 | Detailed of Solutions of CSE-Prelims:2018 • Paper-1 Analysis of 2018 Prelims: 1. The question paper was difficult

Detailed of Solutions of CSE-Prelims:2018 • Paper-1 | 25

37

Consider the following statements :

1. Most of the world’s coral reefs are in tropicalwaters.2. More than one-third of the world’s coral reefsare located in the territories of Australia, Indonesiaand Philippines.3. Coral reefs host far more number of animalphyla than those hosted by tropical rainforests.

Which of the statements given above is/arecorrect ?(a) 1 and 2 only(b) 3 only(c) 1 and 3 only(d) 1, 2 and 3

d Ecology and Environment

Traditional Factual

Aquatic Ecosyst

ems

Statement 1 is correct: Most of theworld's coral reefs are in tropicalwaters.Statement 2 is correct: More thanone-third of the world's coral reefsare located in the territories ofAustralia (17.22%), Indonesia(17.95%) and Philippines (8.81%).Statement 3 is correct: Theseecosystems are packed with thehighestdensities of animals to be foundanywhere on the planet.

http://coral.unep.ch/atlaspr.htmhttp://fnad.org/Documentos/worldatlasofcora01spal.pdf

Difficult

38

“Momentum for Change: Climate Neutral Now” isan initiative launched by

(a) The Intergovernmental panel on ClimateChange(b) The UNEP Secretariat(c) The UNFCCC Secretariat(d) The World Meteorological Organization

c Ecology and Environment Pure Current

International

Institutions/Partnership

s

The UNFCCC secretariat launchedits Climate Neutral Now initiative in2015.

https://unfccc.int/climate-action/momentum-for-change/climate-neutral-now

Medium

39

With reference to educational institutions duringcolonial rule in India, consider the following pairs:

Institution Founder1. Sanskrit College at Benaras WilliamJones2. Calcutta Madarsa WarrenHastings3. Fort William College ArthurWellesley

Which of the pairs given above is/are correct ?(a) 1 and 2(b) 2 only(c) 1 and 3(d) 3 only

bModern Indian History

Traditional Factual

Modern Indian History

Calcutta Madrassa was founded byWarren Hastings(1781), the AsiaticSociety of Bengal founded byWilliam Jones (1784) and theSanskrit College at Benaras foundedby Jonathan Duncan (1794). Therewas a strong urge to make localBritish administrators familiar withIndian culture and tradition. FortWilliam College founded by LordWellesley in 1801 to train the youngBritish recruits to the civil service inIndia was meant to serve primarilythis purpose. Note that here thename given is Arthur Wellesly, whichis wrong as Richard Wellesley wasGovernor-General (1798-1805).

http://www.nios.ac.in/media/documents/SrSec315NEW/History_Module3.pdf

http://ncert.nic.in/ncerts/l/hess102.pdf

Difficult

Page 26: Detailed Solutions CSE-PRELIMS 8 • PAPER-1 · 2018-06-07 · 2 | Detailed of Solutions of CSE-Prelims:2018 • Paper-1 Analysis of 2018 Prelims: 1. The question paper was difficult

26 | Detailed of Solutions of CSE-Prelims:2018 • Paper-1

40

Consider the following pairs :

Regions sometimes mentioned in newsCountry1. CataloniaSpain2. CrimeaHungary3. MindanaoPhilippines4. OromiaNigeria

Which of the pairs given, above are correctlymatched ?(a) 1, 2 and 3(b) 3 and 4 only(c) 1 and 3 only(d) 2 and 4 only

c Geography Pure Current Mapping

Catalonia recently voted to seperatefrom Spain.Crimea was in news because ofRussian annexation of Crimea fromUkraine. For this purpose areferendum was conducted in 2014.Mindanao is in Phlippines.Oromia is in Nigeria.

http://www.thehindu.com/opinion/lead/why-russia-needs-crimea/article5792952.ecehttp://www.thehindu.com/news/international/all-you-need-to-know-about-catalonias-independence-referendum/article18984755.ece

Easy

41

With reference to the Indian Regional NavigationSatellite System (IRNSS), consider the followingstatements :

1. IRNSS has three satellites in geostationary andfour satellites in geosynchronous orbits.2. IRNSS covers entire India and about 5500 sq.km beyond its borders.3. India will have its own satellite navigationsystem with full global coverage by the middle of2019.

Which of the statements given above is/arecorrect?(a) 1 only(b) 1 and 2 only(c) 2 and 3 only(d) None

a Science and Technology

Current Applied

Space Technnology

Statement 1 is correct: Three satellites are located in suitableorbital slots in the geostationary orbitand the remaining four are located ingeosynchronous orbits with therequired inclination and equatorialcrossings in two different planes. Statement 2 is not correct: It isdesigned to provide accurateposition information service to usersin India as well as the regionextending up to 1500 km from itsboundary, which is its primaryservice area.Statement 3 is not correct: Thecoverage of IRNSS is not global butregional.

https://www.isro.gov.in/irnss-programmehttp://www.newindianexpress.com/cities/bengaluru/2018/apr/13/navic-to-cover-indian-mainland-and-1500-km-beyond-countrys-borders-1800886.html

Medium

Page 27: Detailed Solutions CSE-PRELIMS 8 • PAPER-1 · 2018-06-07 · 2 | Detailed of Solutions of CSE-Prelims:2018 • Paper-1 Analysis of 2018 Prelims: 1. The question paper was difficult

Detailed of Solutions of CSE-Prelims:2018 • Paper-1 | 27

42

Consider the following phenomena :

1. Light is affected by gravity.2. The Universe is constantly expanding.3. Matter warps its surrounding space-time.

Which of the above is/are theprediction/predictions of Albert Einstein’s GeneralTheory of Relativity, often discussed in media?(a) 1 and 2 only(b) 3 only(c) 1 and 3 only(d) 1, 2 and 3

d Science and Technology

Current Applied

General Science

Statement 1 is correct: Light isaffected by gravity. Statement 2 is correct: Astronomers using NASA’s HubbleSpace Telescope have discoveredthat the universe is expanding 5percent to 9 percent faster thanexpected. For how fast the universeexpands with time, is called theHubble constant.Statement 3 is correct: Almost allthe warpage in our solar system isproduced directly by matter — theEarth’s matter, the Sun’s matter, thematter of the other planets.

Statement 1: http://curious.astro.cornell.edu/about-us/140-physics/the-theory-of-relativity/general-relativity/1021-if-photons-have-zero-mass-why-do-they-feel-the-effects-of-gravity-advancedStatement 2: https://www.nasa.gov/feature/goddard/2016/nasa-s-hubble-finds-universe-is-expanding-faster-than-expectedhttps://www.its.caltech.edu/~kip/index.html/PubScans/VI-47.pdfStatement 3 : https://www.its.caltech.edu/~kip/index.html/PubScans/VI-47.pdf

Medium

43

With reference to the Genetically Modifiedmustard (GM mustard) developed in India,consider the following statements :

1. GM mustard has the genes of a soil bacteriumthat give the plant the property of pest-resistanceto a wide variety of pests.2. GM mustard has the genes that allow the plantcross-pollination and hybridization.3. GM mustard has been developed jointly by theIARI and Punjab AgricuItal University.

Which of the statements given above is/arecorrect?(a)1 and 3 only(b) 2 only(c)2 and 3(d)1, 2 and 3

b Ecology and Environment

Current Applied

Biotechnology

GM mustard has been developed bya team of scientists at DelhiUniversity led by former vice-chancellor Deepak Pental under agovernment-funded project. Inessence, it uses a system of genesfrom soil bacterium that makesmustard — generally a selfpollinating plant — better suited tohybridisation than current methods.

http://envfor.nic.in/sites/default/files/press-releases/Final_FAQs_GMM_website1.pdf http://www.ifpri.org/blog/let-dhara-flow-gm-mustard-pro-farmer-and-pro-sciencehttp://www.thehindu.com/news/national/GM-mustard-moves-closer-to-approval/article14589131.ece

Medium

Page 28: Detailed Solutions CSE-PRELIMS 8 • PAPER-1 · 2018-06-07 · 2 | Detailed of Solutions of CSE-Prelims:2018 • Paper-1 Analysis of 2018 Prelims: 1. The question paper was difficult

28 | Detailed of Solutions of CSE-Prelims:2018 • Paper-1

44

Consider the following pairs :

Terms sometimes seen in news Context/Topic1. Belle II experiment ArtificialIntelligence2. Blockchain technology Digital/Cryptocurrency3. CRISPR — Cas9 ParticlePhysics

Which of the pairs given above is/are correctlymatched?(a) 1 and 3 only(b) 2 only(c) 2 and 3 only(d) 1, 2 and 3

b Science and Technology

Current Applied

Science and

Technology

Pair 1 is not correct: The Belle IIexperiment is a particle physicsexperiment designed to study theproperties of B mesons Pair 2 iscorrect: Blockchain technology,uses decentralised ledger whichhas application in the crypto-currency Pair 3 is not correct: CRISPR-Cas9is genome editing system.It canremove, add or alter specific DNAsequences in thegenome of higher organisms

https://www.sciencedirect.com/science/article/pii/S240560141500108Xhttps://www.belle2.org/http://www.idrbt.ac.in/assets/publications/Best%20Practices/BCT.pdfhttps://ghr.nlm.nih.gov/primer/genomicresearch/genomeediting

Easy

45

Which of the following statements best describes“carbon fertilization” ?

(a) Increased plant growth due to increasedconcentration of carbon dioxide in the atmosphere(b) Increased temperature of Earth due toincreased concentration of carbon dioxide in theatmosphere(c) Increased acidity of oceans as a result ofincreased concentration of carbon dioxide in theatmosphere(d) Adaptation of all living beings on Earth to theclimate change brought about by the increasedconcentration of carbon dioxide in theatmosphere.

a Ecology and Environment

Traditional Conceptual Ecology

The carbon fertilization effect (CFE)is in principle simple: the largeramount of carbon dioxide in theatmosphere that has resulted fromrising anthropogenic emissionsshould help the growth of plants,which use carbon dioxide duringphotosynthesis. The effect ought toincrease crop yields – and that issome good news for farmers, amidthe overwhelmingly gloomy forecastsfor other aspects of climate change.

https://www.nasa.gov/feature/goddard/2016/carbon-dioxide-fertilization-greening-earth

http://environmentalresearchweb.org/cws/article/news/54347

Easy

Page 29: Detailed Solutions CSE-PRELIMS 8 • PAPER-1 · 2018-06-07 · 2 | Detailed of Solutions of CSE-Prelims:2018 • Paper-1 Analysis of 2018 Prelims: 1. The question paper was difficult

Detailed of Solutions of CSE-Prelims:2018 • Paper-1 | 29

46

When the alarm of your smartphone rings in themorning, you wake up and tap it to stop the alarmwhich causes your geyser to be switched onautomatically. The smart mirror in your bathroomshows the day’s weather and also indicates thelevel of water in your overhead tank. After youtake some groceries from your refrigerator formaking breakfast, it recognises the shortage ofstock in it and places an order for the supply offresh grocery items. When you step’ out of yourhouse and lock the door, all lights, fans, geysersand AC machines get switched off automatically.On your way to office, your car warns you abouttraffic congestion ahead and suggests analternative route, and if you are late for a meeting,it sends a message to your office accordingly.

In the context of emerging communicationtechnologies, which one of the following termsbest applies to the above scenario?(a) Border Gateway Protocol(b) Internet of Things(c) Internet Protocol(d) Virtual Private Network

b Science and Technology

Current Applied

Information

Technology

The internet of things (IoT) is acomputing concept that describesthe idea of everyday physical objectsbeing connected to the internet andbeing able to identify themselves toother devices. The term is closelyidentified with RFID as the method ofcommunication, although it also mayinclude other sensor technologies,wireless technologies or QR codes.The IoT is significant because anobject that can represent itselfdigitally becomes something greaterthan the object by itself. No longerdoes the object relate just to its user,but it is now connected tosurrounding objects and databasedata. When many objects act inunison, they are known as having"ambient intelligence."

https://www.cisco.com/c/dam/en_us/solutions/trends/iot/introduction_to_IoT_november.pdfhttps://www.gsma.com/iot/wp-content/uploads/2014/08/cl_iot_wp_07_14.pdfhttps://www.techopedia.com/definition/28247/internet-of-things-iot

Easy

Page 30: Detailed Solutions CSE-PRELIMS 8 • PAPER-1 · 2018-06-07 · 2 | Detailed of Solutions of CSE-Prelims:2018 • Paper-1 Analysis of 2018 Prelims: 1. The question paper was difficult

30 | Detailed of Solutions of CSE-Prelims:2018 • Paper-1

47

With reference to solar power production in India,consider the following statements :

1. India is the third largest in the world in themanufacture of silicon wafers used in photovoltaicunits.2. The solar power tariffs are determined by theSolar Energy Corporation of India.

Which of the statements given above is/arecorrect ?(a) 1 only(b) 2 only(c) Both 1 and 2(d) Neither 1 nor 2

dEconomic and Social

Development

Current Applied Energy

Both the given statements are notcorrect: . India is not among the topthree prducers in the world in themanufacture of silicon wafers used inphotovoltaic units. The top producersare- China, Russia, USA, Norway,France, Brazil, South Africa, Icelandetc. Solar Energy Corporation of IndiaLtd. (SECI) is a CPSU under theadministrative control of the Ministryof New and Renewable Energy(MNRE) .The company isresponsible for implementation of anumber of schemes of MNRE, majorones being the VGF schemes forlarge-scale grid-connected projectsunder JNNSM, solar park schemeand grid-connected solar rooftopscheme. SECI is not involved indetermining solar power tariffs.

http://seci.gov.in/upload/uploadfiles/files/FAQ.pdf

https://www.statista.com/statistics/268108/world-silicon-production-by-country/

Difficult

48

The staple commodities of export by the EnglishEast India Company from Bengal in the middle ofthe 18th century were

(a) Raw cotton, oil-seeds and opium(b) Sugar, salt, zinc and lead(c) Copper, silver, gold, spices and tea(d) Cotton, silk, saltpetre and opium

dModern Indian History

Traditional Factual

Modern Indian History

India of the 18th century failed tomake progress economically,socially or culturally at an adequatepace. India also exported raw silkand silk fabrics, hardware, indigo,saltpetre, opium, rice, wheat, sugar,pepperand other spices, precious stonesand drugs.

OLD NCERT-INDIAN MODERN HISTORY Difficult

Page 31: Detailed Solutions CSE-PRELIMS 8 • PAPER-1 · 2018-06-07 · 2 | Detailed of Solutions of CSE-Prelims:2018 • Paper-1 Analysis of 2018 Prelims: 1. The question paper was difficult

Detailed of Solutions of CSE-Prelims:2018 • Paper-1 | 31

49

Which one of the following is a very significantaspect of the Champaran Satyagraha ?

(a) Active all-India participation of lawyers,students and women in the National Movement(b) Active involvement of Dalit and Tribalcommunities of India in the National Movement(c) Joining of peasant unrest to India’s NationalMovement(d) Drastic decrease in the cultivation of plantationcrops and commercial crops

cModern Indian History

Traditional Factual

Modern Indian History

Gandhi's first great experiment insatyagraha came in 1917 inChamparan, a district in Bihar. ThePeasantry on the indigo plantationsin the district was excessivelyoppressed by the Europeanplanters. They were compelled togrew indigo on at least 3/20th of theirland and to sell it atprices fixed by the plantern. Similarconditions had prevailed earlier inBengal, but as a result ofa major uprising during 1859-61 thepeasants there had won the freedomfrom the indigo planters.The Champaran satyagraha was thefirst intersection of peasant unrestand the national movement.

http://indianexpress.com/article/opinion/columns/celebrating-champaran-satyagraha-of-1917-british-rule-indian-freedom-movement-4628343/

Medium

50

Who among the following were the founders of the“Hind Mazdoor Sabha” established in 1948 ?

(a) B. Krishna Pillai, E.M.S. Namboodiripad andK.C. George(b) Jayaprakash Narayan, De Dayal Upadhyayand M.N. Roy(c) C.P. Ramaswamy Iyer, K. Kamaraj andVeeresalingam Pantulu(d) Ashok Mehta, T.S. Ramanujam and G.G.Mehta

dPost

Independence

Traditional Factual

Personalities of Freedo

m Struggle

HMS was founded in Calcutta in1948. T The Founding Conferenceelected Com. R.S. Ruikar as the firstPresident, Com. Ashok Mehta as theGeneral Secretary and Com. G.G.Mehta and V.S. Mathur asSecretaries. The members of theWorking Committee included veteranleaders like - Jayaprakash Narayan,V.G. Dalvi, Ms. Aruna Asaf Ali, V.B.Karnik, Dinkar Desai, N.V. Phadke,M.V. Donde, Rajani Mukherjee,Haren Ghosh, Anthoni Pillai, P.S.Chinnadurai, Peter Alwares, A.M.Williams, Munshi Ahmed Din,Vinayak Kulkarni, Nibran Ch. Boraand Basawan Singh.

http://www.hindmazdoorsabha.com/about-hms.php Difficult

Page 32: Detailed Solutions CSE-PRELIMS 8 • PAPER-1 · 2018-06-07 · 2 | Detailed of Solutions of CSE-Prelims:2018 • Paper-1 Analysis of 2018 Prelims: 1. The question paper was difficult

32 | Detailed of Solutions of CSE-Prelims:2018 • Paper-1

51

With reference to the religious practices in India,the “Sthanakvasi” sect belongs to

(a) Buddhism(b) Jainism(c) Vaishnavism(d) Shaivism

b Art and Culture

Traditional Factual Culture

Sthanakvasi arose as reformer of anolder reforming sect, viz., the Lonkasect of Jainism. This Lonka sect wasfounded in about 1474 A.D. byLonkashaha, a rich and well-readmerchant of Ahmedabad. The mainprinciple of this sect was not topractice idol-worship.

http://indianexpress.com/article/india/india-others/the-rite-death-santhara-jain/

Difficult

52

With reference to the cultural history of India,consider the following statements :

1. White marble was used in making BulandDarwaza and Khankah at Fatehpur Sikri.2. Red sandstone and marble were used inmaking Bara Imambara and Rumi Darwaza atLucknow.

Which of the statements given above is/arecorrect?(a) 1 only(b) 2 only(c) Both 1 and 2(d) Neither 1 nor 2

a Art and Culture

Traditional Factual

Architecture

Statement 1 is correct :Buland Darwaza incorporates almost all theessential features of Akbar’sarchitectural traditions: redsandstone, stone carvings, and reliefby inserting white marble. White marble was used in makingboth Buland Darwaza and Khankahat Fatehpur Sikri.Statement 2 is not correct : Thematerial used to raise the mammothstructure of Bada Imambara wasvaried such as lakhauri brick, theunique masala comprising surkhi,choona, urad daal, sheera(molasses), choone ka pani or farez(a natural fevicol like substancederived from a tree). But readstonewas not used in it.

http://www.thehindu.com/opinion/columns/buland-darwaza-and-rumi-darwaza-gateways-to-heaven/article19700177.ece

https://timesofindia.indiatimes.com/city/lucknow/What-makes-Imambara-a-green-building/articleshow/47089758.cms

Difficult

53

Which one of the following foreign travellerselaborately discussed about diamonds anddiamond mines of India?

(a) Francois Bernier(b) Jean-Baptiste Tavernier(c) Jean de Thevenot(d) Abbe Barthelemy Carre

b History Traditional Factual

Modern Indian History

Tavernier was a diamond dealer whohad written on the diamond mines inthe Kingdom of Golconda.

http://ncert.nic.in/ncerts/l/gess106.pdf http://shodhganga.inflibnet.ac.in/bitstream/10603/823/9/09_chapter%203.pdf

http://thefrenchblue.com/timeline2.htm

Difficult

Page 33: Detailed Solutions CSE-PRELIMS 8 • PAPER-1 · 2018-06-07 · 2 | Detailed of Solutions of CSE-Prelims:2018 • Paper-1 Analysis of 2018 Prelims: 1. The question paper was difficult

Detailed of Solutions of CSE-Prelims:2018 • Paper-1 | 33

54

With reference to Indian history, who among thefollowing is a future Buddha, yet to come to savethe world?

(a) Avalokiteshvara(b) Lokesvara(c) Maitreya(d) Padmapani

c Art and Culture

Traditional Factual Culture Maitreya in Buddhist tradition is the

future Buddha.

https://www.britannica.com/topic/Maitreya-BuddhismAL Basham's "Wonder That Was India"ChapterReligion: cults, doctrines and metaphysics

Difficult

55

Which one of the following statements does notapply to the system of Subsidiary Allianceintroduced by Lord Wellesley?

(a) To maintain a large standing army at other’sexpense(b) To keep India safe from Napoleonic danger(c) To secure a fixed income for the Company(d) To establish British paramountcy over theIndian States

cModern Indian History

Traditional Conceptual

Modern Indian History

Lord Wellesley arrived as governorgeneral in 1798, with a dream ofconquest and a lust for personalglory. Napoleonic invasion of Egyptin the summer of 1798 offered him auseful tool to soften London'sresistance to expansion.To assuageLondon's concerns he evolved thepolicy of 'Subsidiary Alliance', whichwould establish control over theinternal affairsof an Indian state. All those whoentered into such an alliance with theBritish had to accept certain termsand conditions:(a) The British would be responsiblefor protecting their ally from externaland internal threats to their power.(b) In the territory of the ally, a Britisharmed contingent would bestationed.(c) The ally would have to providethe resources for maintaining thiscontingent.(d) The ally could enter intoagreements with other rulers orengage in warfare only with thepermission of the British.

.However,the above provisions didn'tmention about a fixed incomesecured for the company.Hence,statement (c) is not correct.

http://ncert.nic.in/ncerts/l/hess102.pdfhttp://ncert.nic.in/ncerts/l/lehs302.pdf

Plassey to partition Page No. 51

Difficult

Page 34: Detailed Solutions CSE-PRELIMS 8 • PAPER-1 · 2018-06-07 · 2 | Detailed of Solutions of CSE-Prelims:2018 • Paper-1 Analysis of 2018 Prelims: 1. The question paper was difficult

34 | Detailed of Solutions of CSE-Prelims:2018 • Paper-1

56

Which one of the following statements correctlydescribes the meaning of legal tender money ?(a) The money which is tendered in courts of lawto defray the fee of legal cases(b) The money which a creditor is undercompulsion to accept in settlement of his claims(c) The bank money in the form of cheques, drafts,bills of exchange, etc.(d) The metallic money in circulation in a country

bEconomic and Social

Development

Traditional Conceptual Banking

Legal tender is any official mediumof payment recognized by law thatcan be used to extinguish a public orprivate debt, or meet a financialobligation. The national currency islegal tender in practically everycountry. A creditor is obligated toaccept legal tender towardrepayment of a debt.

https://www.investopedia.com/terms/l/legal-tender.aspXII: Introductory Marcoeconomics, Chapter3

Easy

57

If a commodity is provided free to the public by theGovernment, then

(a) the opportunity cost is zero.(b) the opportunity cost is ignored.(c) the opportunity cost is transferred from theconsumers of the product to the tax-payingpublic.(d) the opportunity cost is transferred from theconsumers of the product to the Government.

dEconomic and Social

Development

Traditional Conceptual

Microeconomic

s

Opportunity cost is defined as the'value of the best alternative that isforegone when a particular course ofaction is undertaken'. If a commodityis provided free to the public by theGovernment, then the opportunitycost is transferred from theconsumers of the product to theGovernment. Opportunity cost involves themeasurement of sacrifices.When thegovernment provides commodity freeto public, it sacrifices a part of itsresources which could have beenused for second best alternative.

https://www.investopedia.com/terms/o/opportunitycost.asphttps://www.investopedia.com/terms/s/subsidy.asp

Difficult

58

Increase in absolute and per capita real GNP donot connote a higher levelof economic development, if(a) industrial output fails to keep pace withagricultural output.(b) agricultural output fails to keep pace withindustrial output.(c) poverty and unemployment increase.(d) imports grow faster than exports.

cEconomic and Social

Development

Traditional Conceptual

Macroeconomi

cs

An increase in poverty andunemployment cannot result ineconomic development.

https://www.investopedia.com/articles/economics/12/okuns-law.asp Medium

Page 35: Detailed Solutions CSE-PRELIMS 8 • PAPER-1 · 2018-06-07 · 2 | Detailed of Solutions of CSE-Prelims:2018 • Paper-1 Analysis of 2018 Prelims: 1. The question paper was difficult

Detailed of Solutions of CSE-Prelims:2018 • Paper-1 | 35

59

Consider the following statements :Human capital formation as a concept is betterexplained in terms of a processwhich enables

1. individuals of a country to accumulate morecapital.2. increasing the knowledge, skill levels andcapacities of the people of the country.3. accumulation of tangible wealth.4. accumulation of intangible wealth.

Which of the statements given above is/arecorrect?(a) 1 and 2(b) 2 only(c) 2 and 4(d) 1, 3 and 4

cEconomic and Social

Development

Traditional Conceptual

Microeconomic

s

Physical capital is tangible and canbe easily sold in the market like anyother commodity. Human capital isintangible; it is endogenously built inthe body and mind of its owner.The economists sub-divide intangiblecapital into two major categories:‘education’ (consisting of humancapital, including raw labor plus thesum of knowledge, skills and knowhow of the population); and ‘socialcapital’ (represented by beliefs,levels of trust, attitudes, behaviorsand the quality of formal and informalinstitutional infra-structure—includingstability, transparency and otherelements). Thus both statement 2 andstatement 4 explain human capitalformation.

http://ncert.nic.in/ncerts/l/keec105.pdfhttp://developeconomies.com/development-economics/the-intangible-wealth-of-nations/http://econlog.econlib.org/archives/2005/12/intangible_weal_1.htmlhttps://difficultrun.nathanielgivens.com/2018/04/14/total-and-intangible-wealth-world-bank-report-2018/https://www.americanambassadors.org/publications/ambassadors-review/fall-2008/quantifying-the-wealth-of-nations-the-impact-of-intangible-capital-and-implications-for-policy-forumulation

Difficult

60

Despite being a high saving economy, capitalformation may not result insignificant increase in output due to

(a) weak administrative machinery(b) illiteracy(c) high population density(d) high capital-output ratio

dEconomic and Social

Development

Traditional Conceptual

Basic Concept

s of Econom

y

Capital output ratio is the amount ofcapital needed to produce one unit ofoutput. Hence, a higher capital–output ratio reflects inefficiency inthe economy.It is a quantified term whichnecessarily implies reduction inoutput. Other options may/ may notlead to reduction in output.

https://www.businesstoday.in/union-budget-2018-19/columns/analyzing-budget-2018-and-the-saving-investment-climate/story/270422.html

Medium

Page 36: Detailed Solutions CSE-PRELIMS 8 • PAPER-1 · 2018-06-07 · 2 | Detailed of Solutions of CSE-Prelims:2018 • Paper-1 Analysis of 2018 Prelims: 1. The question paper was difficult

36 | Detailed of Solutions of CSE-Prelims:2018 • Paper-1

61

After the Santhal Uprising subsided, whatwas/were the measure/measurestaken by the colonial government ?

1. The territories called `Santhal Paraganas’ werecreated.2. It became illegal for a Santhal to transfer land toa non-Santhal.

Select the correct answer using the code givenbelow :(a) 1 only(b) 2 only(c) Both 1 and 2(d) Neither 1 nor 2

cModern Indian History

Traditional Factual

Tribal and

Peasants

Uprising

British introduced a special systemof administration and for this ActXXXVII of 1855 was passed tosuppress this tribal dominatedpolitical unrest rather than going toroot to solve the problem.Statement 1 is correct :A separatedistrict of Santhal Parganas wasformed.Statement 2 is correct : Land transfer was declared illegal as pertenancy law.

http://yojana.gov.in/tribal-land-rights.asphttp://indianexpress.com/article/india/politics/explained-the-law-political-stakes/

Difficult

62

Economically, one of the results of the British rulein India in the 19thcentury was the

(a) increase in the export of Indian handicrafts(b) growth in the number of Indian owned factories(c) commercialization of Indian agriculture(d) rapid increase in the urban population

cModern Indian History

Traditional Conceptual

Impacts of

British Rule

Commercialisation of agriculturestarted when industrial develpmenttook pace in England. Moreover itbecame prominent around 1860sespecially in cotton and indigo .

Modern India NCERT by Bipin Chandra Easy

63

If the President of India exercises his power asprovided under Article 356 of the Constitution inrespect of a particular State, then

(a) the Assembly of the State is automaticallydissolved.(b) the powers of the Legislature of that State shallbe exercisable by or under the authority of theParliament.(c) Article 19 is suspended in that State.(d) the President can make laws relating to thatState.

b Polity and Governance

Traditional Conceptual

President

When the President’s Rule isimposed in a state the Presidentacquires the extraordinary power todeclare that the powers of the statelegislature are to be exercised by theParliament .

Laxmikanth, 5th edition Page 16.6 Easy

Page 37: Detailed Solutions CSE-PRELIMS 8 • PAPER-1 · 2018-06-07 · 2 | Detailed of Solutions of CSE-Prelims:2018 • Paper-1 Analysis of 2018 Prelims: 1. The question paper was difficult

Detailed of Solutions of CSE-Prelims:2018 • Paper-1 | 37

64

Consider the following pairs : Craft Heritage of1. Puthukkuli shawls Tamil Nadu2. Sujni embroidery Maharashtra3. Uppada Jamdani Saris Karnataka

Which of the pairs given above is/are correct ?(a) 1 only(b) 1 and 2(c) 3 only(d) 2 and 3

a Art and Culture

Traditional Factual Crafts

Pair one is correctly matched :Puthukkuli shawls are made byTodas of Nilgiri hills in Tamil Nadu. Itis locally called Pugur, meaningflower, the fine and intricate Todaembroidery is done by tribal men andwomen on shawls. Pair 2 and 3 are not correct:Uppada Sarees are a distinctivetypes of sarees and dress materialswoven in Uppada town of AndhraPradesh.Sujani (or Sujini) is one of the mostpopular form of conventional art andcraft prevailing in Bihar.

http://www.lepakshihandicrafts.gov.in/category-uppada.htmlhttp://www.umsas.org.in/en/bihar-arts-crafts/sujini-embroidery/

Difficult

65

In which of the following areas can GPStechnology be used ?

1. Mobile phone operations2. Banking operations3. Controlling the power grids

Select the correct answer using the code givenbelow :(a) 1 only(b) 2 and 3 only(c) 1 and 3 only(d) 1, 2 and 3

d Science and Technology

Traditional Conceptual

Techonology

It can be used in all the threementioned opertations :1.Cell phones with GPS receiverscommunicate with units from amongthe 30 global positioning satellites inthe GPS system. This has been usedfor tracking and securitymeasurements.2.In the banking industry, live GPStracking devices are placed on cashbundles or boxes so that money canbe traced to and from armouredvehicles and vaults.3.Power system owners andoperators use GPS for navigation,position and timing

https://www.sciencedirect.com/science/article/pii/S1877042813024397https://www.brighthub.com/electronics/gps/articles/93343.aspxhttps://www.gps.gov/cgsic/meetings/2016/silverstein.pdf

Easy

Page 38: Detailed Solutions CSE-PRELIMS 8 • PAPER-1 · 2018-06-07 · 2 | Detailed of Solutions of CSE-Prelims:2018 • Paper-1 Analysis of 2018 Prelims: 1. The question paper was difficult

38 | Detailed of Solutions of CSE-Prelims:2018 • Paper-1

66

Consider the following statements :

1. The Reserve Bank of India manages andservices Government of India Securities but notany State Government Securities.2. Treasury bills are issued by the Government ofIndia and there are no treasury bills issued by theState Governments.3. Treasury bills offer are issued at a discountfrom the par value.

Which of the statements given above is/arecorrect?(a) 1 and 2 only(b) 3 only(c) 2 and 3 only(d) 1, 2 and 3

cEconomic and Social

Development

Traditional Conceptual Banking

statement 2 and 3 are correct butstatement 1 is not correct asexplained below.RBI manages G-Secs of both theUnion Government and the stategovernments.In India, the Central Governmentissues both, treasury bills and bondsor dated securities while the StateGovernments issue only bonds ordated securities, which are called theState Development Loans (SDLs).Treasury bills are zero couponsecurities and pay no interest. Theyare issued at a discount andredeemed at the face value atmaturity. For example, a 91 dayTreasury bill of ₹100/- (face value)may be issued at say ₹ 98.20, that is, at a discount of say, ₹1.80 and wouldbe redeemed at the face value of₹100/-.

https://m.rbi.org.in/scripts/FAQView.aspx?Id=79 Difficult

67

Consider the following statements :

1. The Earth’s magnetic field has reversed everyfew hundredthousand years.2. When the Earth was created more than 4000million years ago, there was 54% oxygen and nocarbondioxide.3. When living organisms originated, they modifiedthe early atmosphere of the Earth.

Which of the statements given above is/arecorrect ?(a) 1 only(b) 2 and 3 only(c) 1 and 3 only(d) 1, 2 and 3

c Geography Traditional Conceptual

Physical Geogra

phy

Both the statement 1 and 3 arecorrect but statement 2 is notcorrect.The early atmosphere wasdominated by hydrogen and helium.There are three stages in theevolution of the present atmosphere.The first stage ismarked by the loss of primordialatmosphere.In the second stage, thehot interior of the earth contributed tothe evolution of the atmosphere.Finally, the composition of theatmosphere was modified by theliving world through the process ofphotosynthesis

https://www.nasa.gov/topics/earth/features/2012-poleReversal.html

http://ncert.nic.in/ncerts/l/kegy202.pdf

Medium

Page 39: Detailed Solutions CSE-PRELIMS 8 • PAPER-1 · 2018-06-07 · 2 | Detailed of Solutions of CSE-Prelims:2018 • Paper-1 Analysis of 2018 Prelims: 1. The question paper was difficult

Detailed of Solutions of CSE-Prelims:2018 • Paper-1 | 39

68

The terms ‘WannaCry, Petya and EternalBlue’sometimes mentioned inthe news recently are related to(a) Exoplanets(b) Cryptocurrency(c) Cyber attacks(d) Mini satellites

c Science and Technology Pure Current Techon

ology

WannaCry and Petya werecyberattacks in form of ransomwarethat were in the news last year.EternalBlue was an exploit in older

Windows systems released by TheShadowBrokers a few months prior to theattack.

http://www.thehindu.com/todays-paper/tp-opinion/held-to-ransomware/article19167509.ece

Easy

69

With reference to the circumstances in Indianagriculture, the concept of“Conservation Agriculture” assumes significance.Which of the following fallunder the Conservation Agriculture ?

1. Avoiding the monoculture practices2. Adopting minimum tillage3. Avoiding the cultivation of plantation crops4. Using crop residues to cover soil surface5. Adopting spatial and temporal cropsequencing/crop rotations

Select the correct answer using the code givenbelow :(a) 1, 3 and 4(b) 2, 3, 4 and 5(c) 2, 4 and 5(d) 1, 2, 3 and 5

c Ecology and Environment

Traditional Conceptual

Physical Geogra

phy

Conservation Agriculture is a farmingsystem that promotes maintenanceof a permanent soil cover, minimumsoil disturbance (i.e. no tillage), anddiversification of plant species.Avoiding a practice like monoculture,plantation etc is not a conservationstep. Monoculture practices can beimproved upon to reduce itsecological impact. ConservationAgriculture does not include avoidingthe practice itself.Hence statement 1 and statement 3are not applicable underconservation practice.

http://www.fao.org/3/a-i7480e.pdf Difficult

Page 40: Detailed Solutions CSE-PRELIMS 8 • PAPER-1 · 2018-06-07 · 2 | Detailed of Solutions of CSE-Prelims:2018 • Paper-1 Analysis of 2018 Prelims: 1. The question paper was difficult

40 | Detailed of Solutions of CSE-Prelims:2018 • Paper-1

70

The term “sixth mass, extinction/sixth extinction” isoften mentioned in the news in the context of thediscussion of

(a) Widespread monoculture practices inagriculture and largescale commercial farmingwithindiscriminate use of chemicals in many parts ofthe world that may result in the loss of goodnative ecosystems.(b) Fears of a possible collision of a meteorite withthe Earth in the near future in the manner ithappened 65 million years ago that caused themass extinction of many species including thoseof dinosaurs.(c) Large scale cultivation of genetically modifiedcrops in many parts of the world and promotingtheir cultivation in other parts of the world whichmay cause the disappearance ofgood native crop plants and the loss of foodbiodiversity.(d) Mankind’s over-exploitation/ misuse of naturalresources, fragmentation/loss of naturalhabitats, destruction of ecosystems, pollution andglobal climate change.

d Ecology and Environment

Traditional Factual Ecology

The Holocene extinction,otherwisereferred to as the Sixth extinction orAnthropocene extinction, is theongoing extinction event of speciesduring the present Holocene epoch,mainly as a result of human activity.

http://www.thehindu.com/todays-paper/tp-national/earth-facing-sixth-mass-extinction/article19260792.ece

Medium

Page 41: Detailed Solutions CSE-PRELIMS 8 • PAPER-1 · 2018-06-07 · 2 | Detailed of Solutions of CSE-Prelims:2018 • Paper-1 Analysis of 2018 Prelims: 1. The question paper was difficult

Detailed of Solutions of CSE-Prelims:2018 • Paper-1 | 41

71

Consider the following statements :1. Capital Adequacy Ratio (CAR) is the amountthat banks have to maintain in the form of theirown funds to offset any loss that banks incur if theaccount-holders fail to repay dues.2. CAR is decided by each individual bank

Which of the statements given above is/arecorrect ?(a) 1 only(b) 2 only(c) Both 1 and 2(d) Neither 1 nor 2

aEconomic and Social

Development

Traditional Conceptual

Eonomy Concept

s

Statement 1 is correct : The reasonminimum capital adequacy ratios(CARs) are critical is to make surethat banks have enough cushion toabsorb a reasonable amount oflosses before they become insolventand consequently lose depositors’funds. The capital adequacy ratiosensure the efficiency and stability ofa nation’s financial system bylowering the risk of banks becominginsolvent.Statement 2 is also not correct:Indian scheduled commercial banksare required to maintain a CAR asper the RBI norms.

https://www.investopedia.com/terms/c/capitaladequacyratio.asphttps://economictimes.indiatimes.com/definition/capital-adequacy-ratio

Medium

72

The identity platform Aadhaar’ provides open“Application Programming Interfaces (APIs)”.What does it imply ?

1. It can be integrated into any electronic device.2. Online authentication using iris is possible.

Which of the statements given above is/arecorrect ?(a) 1 only(b) 2 only(c) Both 1 and 2(d) Neither 1 nor 2

c Science and Technology

Current Applied

Techonology

statement 1 is correct: An application programming interface(API) is a set of subroutinedefinitions, protocols, and tools forbuilding application software. Ingeneral terms, it is a set of clearlydefined methods of communicationbetween various softwarecomponents.The API can beintegrated into any electronic device.statement 2 is correct : Iris scan isone of the possible modes of Aadharauthentication.

https://aadhaarapi.com/,https://uidai.gov.in/images/FrontPageUpdates/aadhaar_authentication_api_2_0.pdf

Medium

73

Very recently, in which of the following countrieshave lakhs of people either suffered from severefamine/acute malnutrition or died due to starvationcaused by war/ethnic conflicts ?

(a) Angola and Zambia(b) Morocco and Tunisia(c) Venezuela and Colombia(d) Yemen and South Sudan

d Geography Pure Current Mapping

South Sudan is in the midst of acatastrophic humanitarian crisisdriven by three years of brutal civilwar. Nearly half of the population arefacing extreme hunger and are inneed of urgent aid.Yemen, one of the Arab world'spoorest countries, has beendevastated by a civil war since 2015.

https://www.oxfam.org/en/emergencies/hunger-crisis-south-sudan

http://www.bbc.com/news/world-middle-east-29319423

Easy

Page 42: Detailed Solutions CSE-PRELIMS 8 • PAPER-1 · 2018-06-07 · 2 | Detailed of Solutions of CSE-Prelims:2018 • Paper-1 Analysis of 2018 Prelims: 1. The question paper was difficult

42 | Detailed of Solutions of CSE-Prelims:2018 • Paper-1

74

Regarding Wood’s Dispatch, which of thefollowing statements are true ?

1. Grants-in-Aid system was introduced.2. Establishment of universities wasrecommended.3. English as a medium of instruction at all levelsof education was recommended.

Select the correct answer using the code givenbelow :(a) 1 and 2 only(b) 2 and 3 only(c) 1 and 3 only(d) 1, 2 and 3

aModern Indian History

Traditional Factual

Education

Statement 3 is not correct .As per the Wood's Dispatch,the medium ofinstruction at the primary level was tobe vernacular while at the higherlevels it would be English. As per this despatch:1.An education department was tobe set in every province.2.Universities on the model of theLondon University be established inbig cities such as Bombay, Calcuttaand Madras.3.Affiliated private schools should begiven grant in aid.4.The Indian natives should be giventraining in their mother tongue also

http://www.nios.ac.in/media/documents/dled/Block1_501.pdf

Medium

75

With reference to the Parliament of India, which ofthe following Parliamentary Committeesscrutinizes and reports to the House whether thepowers to make regulations, rules, sub-rules, by-laws, etc. conferred by the Constitution ordelegated by the Parliament are being properlyexercised by the Executive within the scope ofsuch delegation ?

(a) Committee on Government Assurances(b) Committee on Subordinate Legislation(c) Rules Committee(d) Business Advisory Committee

b Polity and Governance

Traditional Factual

Parliamentary

Committees

Amongst the mechanisms evolvedby the legislature to exercise controlover the delegated legislation, themost important is the constitution ofthe Committee on SubordinateLegislation. It is this Committee ofthe legislature which examines if thepowers conferred by the Constitutionor delegated under an Act passed bythe legislature have been dulyexercised and are within theconferment or delegation, and notbeyond. It has to see that delegatedlegislation does not transgress intoareas not prescribed for it, and alsothat it does not venture to intrude into the sphere which is the sole concernof the legislature itself.

https://rajyasabha.nic.in/rsnew/practice_procedure/book13.asp

Medium

Page 43: Detailed Solutions CSE-PRELIMS 8 • PAPER-1 · 2018-06-07 · 2 | Detailed of Solutions of CSE-Prelims:2018 • Paper-1 Analysis of 2018 Prelims: 1. The question paper was difficult

Detailed of Solutions of CSE-Prelims:2018 • Paper-1 | 43

76

Consider the following statements:

1. As per the Right to Education (RTE) Act, to beeligible for appointment as a teacher in a State, aperson would be required to possess the minimumqualification laid down by the concerned StateCouncil of Teacher Education.2. As per the RTE Act, for teaching primaryclasses, a candidate is required to pass a TeacherEligibility Test conducted in accordance with theNational Council of Teacher ducation guidelines.3. In India, more than 90% of teacher educationinstitutions are directly under the StateGovernments.

Which of the statements given above is/arecorrect ?(a) 1 and 2(b) 2 only(c) 1 and 3 only(d) 3 only

bEconomic and Social

Development

Traditional Conceptual

Governance

Statement 1 is not correct: The Central Government has notified theNational Council for eacherEducation (NCTE) as the academicauthority for prescribing teacherqualifications.tatement 2 is correct : As perSection 23(1) of the RTE Act, 2009,the National Council for eacherEducation (NCTE) has laid down theminimum qualifications for a personto be eligible for appointment as ateacher in class I to VIII.One of the essential qualifications fora person to be eligible forappointment as a teacher is thathe/she should pass the TeacherEligibility Test (TET) which will beconducted by the appropriateGovernment.Statement 3 is not correct: Morethan 90% of teacher educationinstitutions are not directly under theState Governments.

http://mhrd.gov.in/sites/upload_files/mhrd/files/upload_document/RTE_Section_wise_rationale_rev_0.pdfhttp://mhrd.gov.in/sites/upload_files/mhrd/files/upload_document/RTE_TET.pdf

Difficult

Page 44: Detailed Solutions CSE-PRELIMS 8 • PAPER-1 · 2018-06-07 · 2 | Detailed of Solutions of CSE-Prelims:2018 • Paper-1 Analysis of 2018 Prelims: 1. The question paper was difficult

44 | Detailed of Solutions of CSE-Prelims:2018 • Paper-1

77

Consider the following pairs: Traditions : State 1. Chapchar Kut festival : Mizoram 2. Khongjom Parba ballad : Manipur 3. Thang-Ta dance : Sikkim

Which of the pairs given above is/are correct ? (a) 1 only (b) 1 and 2 (c) 3 only (d) 2 and 3

b Art and Culture

Traditional Factual

Traditions

Chapchar Kut festival: It is a festivalof Mizoram.The festival is heldduring the period when the bamboosand trees that have been cut downare being awaited to dry to be burntfor jhumming. During this brief layoffperiod of jhumming, the Mizoancestors could have all the time forthemselves. They spend their leisurehunting games, fishing, et al. Khongjom Parba ballad: It is a styleof ballad singing from Manipur thatdepicts stories of the heroic battlefought by Manipuris against Britishforces in 1891.Thang-ta dance: It is a martial danceof Manipur not Sikkim.

http://mizoram.nic.in/about/chapcharkut.htm

http://manipurtimes.com/reviving-the-dying-traditional-manipuri-music-khongjom-parva-padma-shri-awardee-nameirakpam-ibemni/

http://ccrtindia.gov.in/manipuri.php

Difficult

Page 45: Detailed Solutions CSE-PRELIMS 8 • PAPER-1 · 2018-06-07 · 2 | Detailed of Solutions of CSE-Prelims:2018 • Paper-1 Analysis of 2018 Prelims: 1. The question paper was difficult

Detailed of Solutions of CSE-Prelims:2018 • Paper-1 | 45

78

Consider the following statements :

1. The Food Safety and Standards Act, 2006replaced the Prevention of Food Adulteration Act,1954.2. The Food Safety and Standards Authority ofIndia (FSSAI) is under the charge of DirectorGeneral of Health Services in the Union Ministry ofHealth and Family Welfare.

Which of the statements given above is/arecorrect ?(a) 1 only(b) 2 only(c) Both 1 and 2(d) Neither 1 nor 2

a Polity and Governance

Traditional Factual

Governance

Statement 1 is correct: The FoodSafety and Standards Authority ofIndia (FSSAI) has been establishedunder Food Safety and Standards ,2006 which consolidates various acts & orders that have hitherto handledfood related issues in variousMinistries and Departments.The Act consolidates the followingacts and orders:• Prevention of Food AdulterationAct,1954,• Fruit Products Order , 1955, • Meat Food Products Order,1973,• Vegetable Oil Products (Control)Order, 1947,• Edible Oils Packaging(Regulation)Order 1988, • Solvent Extracted Oil, De- OiledMeal and Edible Flour (Control)Order, 1967, -Milk and Milk ProductsOrder, 1992 Statement 2 is not correct: FSSAIis an autonomous body. Ministry ofHealth & Family Welfare,Government of India is theAdministrative Ministry for theimplementation of FSSAI. It is notunder the charge of Director Generalof Health Services.

http://www.fssai.gov.in/home/about-us/introduction.htmlhttp://www.dnaindia.com/business/report-fssai-favours-comprehensive-legislation-for-organic-foods-2597234https://www.fssai.gov.in/dam/jcr:7936048f-5532-403a-add4-10d4e72dd2f5/Circular_FSSAI_Filling_Post_31_08_2017.pdf

Difficult

79

The term “two-state solution” is sometimesmentioned in the news in the context of the affairsof

(a) China(b) Israel(c) Iraq(d) Yemen

b International Relations Pure Current

International

Relations

The two-state solution refers to asolution of the Israeli–Palestinianconflict which calls for "two states fortwo groups of peoplThe two-statesolution refers to a solution of theIsraeli–Palestinian conflict whichcalls for "two states for two groups ofpeople."

https://www.nytimes.com/2016/12/29/world/middleeast/israel-palestinians-two-state-solution.html

Easy

Page 46: Detailed Solutions CSE-PRELIMS 8 • PAPER-1 · 2018-06-07 · 2 | Detailed of Solutions of CSE-Prelims:2018 • Paper-1 Analysis of 2018 Prelims: 1. The question paper was difficult

46 | Detailed of Solutions of CSE-Prelims:2018 • Paper-1

80

With reference to the provisions made under theNational Food Security Act, 2013; consider thefollowing statements :

1. The families coming under the category of‘below poverty line (BPL)’ only are eligible toreceive subsidised food grains.2. The eldest woman in a household, of age 18years or above, shall be the head of thehousehold for the purpose of issuance of a rationcard.3. Pregnant women and lactating mothers areentitled to a ‘takehome ration’ of 1600 calories perday during pregnancy and for six monthsthereafter.

Which of the statements given above is/arecorrect ?(a) 1 and 2(b) 2 only(c) 1 and 3(d) 3 only

b Polity and Governance

Traditional Conceptual

Governance

Statetement 1 is not correct: TheAct defines ‘eligible households’under two categories: (i) householdscovered under the Antyodaya AnnaYojana (AAY); and (ii) householdscovered as the priority householdsunder the Targeted PublicDistributionSystem (TPDS). Statetement 2 is correct: Theeldest woman of eighteen years ofage or above in every eligiblehousehold, wherever available, shallbe head of the household for thepurpose of issuance of ration card. Statetement 3 is not correct: It is600 KCal not 1600 Calories.

https://rajyasabha.nic.in/rsnew/publication_electronic/National_Food_security_Act2013.pdf

Medium

81

India enacted The Geographical Indications ofGoods (Registration and Protection) Act, 1999 inorder to comply with the obligations to (a) ILO(b) IMF(c) UNCTAD(d) WTO

dEconomic and Social

Development

Traditional Factual

Intellectual

Property

India, as a member of the WorldTrade Organization (WTO), enactedthe Geographical Indications ofGoods (Registration &Protection)Act, 1999 has come intoforce with effect from 15thSeptember 2003.

http://www.ipindia.nic.in/about-us-gi.htm Easy

Page 47: Detailed Solutions CSE-PRELIMS 8 • PAPER-1 · 2018-06-07 · 2 | Detailed of Solutions of CSE-Prelims:2018 • Paper-1 Analysis of 2018 Prelims: 1. The question paper was difficult

Detailed of Solutions of CSE-Prelims:2018 • Paper-1 | 47

82

Consider the following statements :

1. In India, State Governments do not have thepower to auction non-coal mines.2. Andhra Pradesh and Jharkhand do not havegold mines.3. Rajasthan has iron ore mines.

Which of the statements given above is/arecorrect ?(a) 1 and 2(b) 2 only(c) 1 and 3(d) 3 only

dEconomic and Social

Development

Traditional Conceptual

Distribution of

Resources

Statement 1 is not correct: Stategovernments have the power toauction non-coal mines. For example-Limestone auctions byChhattisgarh,Andhra Pradesh,Gujarat and others.Statement 2 is not correct:Jharkhand has gold mines likeRungta mines whose auction washeld last year. Statement 3 is correct: Rajasthanhas iron ore mines in Bhilwara.

https://www.financialexpress.com/industry/95-non-coal-mines-may-be-auctioned-in-fy19/1115556/

https://timesofindia.indiatimes.com/city/delhi/Jharkhand-ready-to-auction-gold-mines-next-month/articleshow/51925705.cms

https://community.data.gov.in/working-and-non-working-iron-ore-mines-in-various-states-as-on-20-04-2016/

http://indianexpress.com/article/business/business-others/non-coal-blocks-states-undertake-groundwork-for-auction-of-71-mines-this-year-4525263/

Difficult

83

With reference to digital payments, consider thefollowing statements :

1. BHIM app allows the user to transfer money toanyone with a UPI-enabled bank account.2. While a chip-pin debit card has four factors ofauthentication, BHIM app has only two factors ofauthentication.

Which of the statements given above is/arecorrect ?

(a) 1 only(b) 2 only(c) Both 1 and 2(d) Neither 1 nor 2

a Science and Technology

Current Applied

Technology/Digital India

Statement 1 is correct: BharatInterface for Money (BHIM) is apayment app that lets you makesimple, easy and quick transactionsusing Unified Payments Interface(UPI). You can make direct bankpayments to anyone on UPI usingtheir UPI ID or scanning their QRwith the BHIM app. You can alsorequest money through the app froma UPI ID.Statement 2 is not correct: Three-factor authentication is key safetyfeature on the BHIM app.

https://www.bhimupi.org.in/who-we-are

http://indianexpress.com/article/technology/tech-news-technology/bhim-app-narendra-modi-upi-online-transaction-safety-4453599/

Difficult

Page 48: Detailed Solutions CSE-PRELIMS 8 • PAPER-1 · 2018-06-07 · 2 | Detailed of Solutions of CSE-Prelims:2018 • Paper-1 Analysis of 2018 Prelims: 1. The question paper was difficult

48 | Detailed of Solutions of CSE-Prelims:2018 • Paper-1

84

Among the following cities, which one lies on alongitude closest to that of Delhi?

(a) Bengaluru(b) Hyderabad(c) Nagpur(d) Pune

a Geography Traditional Conceptual

Mapping

The longitudes of the cities are: Delhilongitude:77.1025° E, Bangalore:77.5946° EHyderabad's longitude is 78.4867° ENagpur's longitude is 79.0882° EPune's longitude is 73.8567° E

Atlas Difficult

85

International Labour Organisation’s Conventions138 and 182 are related to(a) Child labour(b) Adaptation of agricultural practices to globalclimate change(c) Regulation of food prices and food security(d) Gender parity at the workplace

a International Relations

Traditional Factual

International

Organization

The two conventions are:ILO Convention No. 182 on the worstforms of child labourILO Convention No. 138 on theminimum age for admission toemployment and work.

http://www.ilo.org/ipec/facts/ILOconventionsonchildlabour/lang--en/index.htm

Easy

Page 49: Detailed Solutions CSE-PRELIMS 8 • PAPER-1 · 2018-06-07 · 2 | Detailed of Solutions of CSE-Prelims:2018 • Paper-1 Analysis of 2018 Prelims: 1. The question paper was difficult

Detailed of Solutions of CSE-Prelims:2018 • Paper-1 | 49

86

Consider the following statements

1. In the first Lok Sabha, the single largest party inthe opposition was the Swatantra Party.2. In the Lok Sabha, a “Leader of the Opposition”was recognised for the first time in 1969.3. In the Lok Sabha, if a party does not have aminimum of 75 members, its leader cannot berecognised as the Leader of the Opposition.

Which of the statements given above is/arecorrect ?(a) 1 and 3 only(b) 2 only(c) 2 and 3 only(d) 1, 2 and 3

b Polity and Governance

Traditional Factual

Lok Sabha

Statement 1 is not Correct: In thefirst Lok Sabha, the single largestparty in the opposition wasCommunist Party of India.Swatantra Party was founded in1959 by C. Rajagopalachari.Heparted ways with the Congress in1957 after being disillusioned by thepath it was taking. He founded theSwatantra Party in 1959, whichfavoured classical liberal principlesand free enterprise.Thus the partydid not participate in the first LokSabha elections.Statement 2 is correct: In the LokSabha, a “Leader of the Opposition”was recognised forthe first time in 1969. Leader ofOppsition is not a constitutionalauthority.Statement 3 is not correct: The options can be easily eliminated bystatement 3 ,which is incorrect. Aminimum of 10% members of thetotal membership of the house isrequired by a party for its leader tobe recognised as Leader ofOpposition.

Indian Polity by Laxmikant

https://www.livemint.com/Sundayapp/XlvTGlfJcdJu9mQGZcksTI/C-Rajagopalachari--Why-Swatantra.html

Easy

Page 50: Detailed Solutions CSE-PRELIMS 8 • PAPER-1 · 2018-06-07 · 2 | Detailed of Solutions of CSE-Prelims:2018 • Paper-1 Analysis of 2018 Prelims: 1. The question paper was difficult

50 | Detailed of Solutions of CSE-Prelims:2018 • Paper-1

87

Which of the following leaf modifications occur(s)in the desert areas to inhibit water loss ?

1. Hard and waxy leaves2. Tiny leaves3. Thorns instead of leaves

Select the correct answer using the code givenbelow.(a) 2 and 3 only(b) 2 only(c) 3 only(d) 1, 2 and 3

d Geography Traditional Conceptual Ecology

Desert plants lose very little waterthrough transpiration. The leaves indesert plants are either absent, verysmall, or they are present in theshape of spines. This helps inreducing loss ofwater from the leaves throughtranspiration.The stem is also covered with a thickwaxy layer, which helps to retainwater.

http://ncert.nic.in/ncerts/l/fesc109.pdf Medium

88

As per the NSSO 70th Round “SituationAssessment - Survey of Agricultural Households”,

consider the following statements :1. Rajasthan has the highest percentage share ofagricultural households among its ruralhouseholds.2. Out of the total agricultural households in thecountry, a little over 60 percent; belong to OBCs.3. In Kerala, a little over 60 percent of agriculturalhouseholds reported to have received maximumincome from sources other than agriculturalactivities.

Which of the statements given above is/arecorrect ?(a) 2 and 3 only(b) 2 only(c) 1 and 3 only(d) 1, 2 and 3

cEconomic and Social

Development

Current Applied

Agriculture

Statement 1 is correct: Among themajor States, Rajasthan had highestpercentage of agriculturalhouseholds (78.4 percent) among itsrural households followed by UttarPradesh (74.8 percent) and MadhyaPradesh (70.8 percent).Statement 2 is not correct: It islittle over 45% not 60%.Statement 3 is Correct: Agriculturalactivity (cultivation, livestock andother agricultural activities) wasreported to be the principal source ofincome for majority of thehouseholds in all the major States,except Kerala where about 61percent of the agriculturalhouseholds reported to have earnedmaximum income from sources otherthan agricultural activities

http://mospi.nic.in/sites/default/files/publication_reports/KI_70_33_19dec14.pdf Difficult

Page 51: Detailed Solutions CSE-PRELIMS 8 • PAPER-1 · 2018-06-07 · 2 | Detailed of Solutions of CSE-Prelims:2018 • Paper-1 Analysis of 2018 Prelims: 1. The question paper was difficult

Detailed of Solutions of CSE-Prelims:2018 • Paper-1 | 51

89

How is the National Green Tribunal (NGT) different from the Central Pollution Control Board (CPCB) ?

1. The NGT has been established by an Actwhereas the CPCB has been created by anexecutive order of the Government.2. The NGT provides environmental justice andhelps reduce the burden of litigation in thehigher courts whereas the CPCB promotescleanliness of streams and wells, and aims toimprove the quality of air in the country.

Which of the statements given above is/arecorrect ?(a) 1 only(b) 2 only(c) Both 1 and 2(d) Neither 1 nor 2

b Ecology and Environment

Traditional Conceptual

Governance/Bo

dies

Statement 1 is not correct: TheCentral Pollution Control Board(CPCB), statutory organisation, wasconstituted in September, 1974under the Water (Prevention andControl of Pollution) Act, 1974.Statement 2 is correct: TheNational Green Tribunal has beenestablished on 18.10.2010 under theNational Green Tribunal Act 2010 foreffective and expeditious disposal ofcases relating to environmentalprotection and conservation offorests and other natural resources.Principal Functions of the CPCB, asspelt out in the Water (Preventionand Control of Pollution) Act, 1974,and the Air (Prevention and Controlof Pollution) Act, 1981, (i) to promote cleanliness of streamsand wells in different areas of theStates by prevention, control andabatement of water pollution, and (ii) to improve the quality of air and toprevent, control or abate air pollutionin the country.

http://cpcb.nic.in/Introduction/ Medium

Page 52: Detailed Solutions CSE-PRELIMS 8 • PAPER-1 · 2018-06-07 · 2 | Detailed of Solutions of CSE-Prelims:2018 • Paper-1 Analysis of 2018 Prelims: 1. The question paper was difficult

52 | Detailed of Solutions of CSE-Prelims:2018 • Paper-1

90

Consider the following statements

1. The Parliament of India can place a particularlaw in the Ninth Schedule of the Constitution ofIndia.2. The validity of a law placed in the NinthSchedule cannot be examined by any court andnojudgement can be made on it

Which of the statements given above is/arecorrect ?(a) 1 only(b) 2 only(c) Both 1 and 2(d) Neither 1 nor 2

a Polity and Governance

Traditional Conceptual Polity

Statement 1 is correct: TheParliament of India can place aparticular law in the ninth schedule ofthe Constitution. Once a law isenacted and included in the NinthSchedule, it gets protection underArticle 31-B (validation of certainActs and Regulations).Statement 2 is not correct: In theCoelho Case Supreme Court Benchheld that all such laws included inthe Ninth Schedule after April 24,1973 would be tested individually onthe touchstone of violation offundamental rights or the basicstructure doctrine. The laws wouldbe examined separately by a three-judge Bench and if these were foundto violate the fundamental rights,abridge or abrogate any of the rightsor protection granted to the peoplewould be set aside.

http://www.thehindu.com/todays-paper/IX-Schedule-laws-open-to-review/article14705323.ece

Medium

91

Which one of the following best describes the term“Merchant Discount Rate” sometimes seen innews ?

(a) The incentive given by a bank to a merchantfor accepting payments through debit cardspertaining to that bank. (b) The amount paid back by banks to theircustomers when they use debit cards for financialtransactions for purchasing goods or services. (c) The charge to a merchant by a bank foraccepting payments from his customers throughthe bank’s debit cards. (d) The incentive given by the Government tomerchants for promoting digital payments by theircustomers through Point of Sale (PoS) machinesand debit cards.

cEconomic and Social

Development

Current Applied Banking

MDR is a fee charged from amerchant by a bank for acceptingpayments from customers throughcredit and debit cards in theirestablishments. MDR compensates the card issuingbank, the lender which puts the PoSterminal and payment gateways suchas Mastercard or Visa for theirservices.

https://economictimes.indiatimes.com/wealth/spend/what-is-merchant-discount-rate/articleshow/62390733.cms

Easy

Page 53: Detailed Solutions CSE-PRELIMS 8 • PAPER-1 · 2018-06-07 · 2 | Detailed of Solutions of CSE-Prelims:2018 • Paper-1 Analysis of 2018 Prelims: 1. The question paper was difficult

Detailed of Solutions of CSE-Prelims:2018 • Paper-1 | 53

92

What is/are the consequence/ consequences of acountry becoming the member of the ‘NuclearSuppliers Group’ ?

1. It will have access to the latest and mostefficient nuclear technologies.2. It automatically becomes a member of “TheTreaty on the Non-Proliferation of NuclearWeapons (NPT)”.

Which of the statements given above is/arecorrect ?(a) 1 only(b) 2 only(c) Both 1 and 2(d) Neither 1 nor 2

a International Relations

Current Applied

International

Institutions/Partnership

s

Statement 1 is correct: The NSG isthe top club of countries whichcontrols access to technology andguards against proliferation. Itsmembership is important for India toaccess cutting-edge high technology.Statement 2 is not correct: Treatyon the Non Proliferation of NuclearWeapons(NPT) is one of theconditions held to join the NSG. Indiawill not become a member of NPT byjoining NSG.

http://indianexpress.com/article/india/indias-membership-to-nsg-delhi-works-on-beijing-to-drop-objections-on-nuclear-suppliers-group-club-entry-5132252/

Easy

93

With reference to India’s decision to levy anequalization tax of 6% on online advertisementservices offered by non-resident entities, which ofthe following statements is/are correct ?

1. It is introduced as a part of the Income Tax Act.2. Non-resident entities that offer advertisementservices in India can claim a tax credit in theirhome country under the “Double TaxationAvoidance Agreements”.

Select the correct answer using the code givenbelow :(a) 1 only(b) 2 only(c) Both 1 and 2(d) Neither 1 nor 2

dEconomic and Social

Development

Current Applied

Fiscal Policy

Statement 1 is not correct : Thelevy was introduced in the Budget aspart of the finance bill and not as apart of Income Tax Act. Statement 2 is not correct: Thecompanies bearing the brunt of thelevy would not be able to enjoy thebenefit of tax treaties to avoid doubletaxation in their home countries.

https://www.business-standard.com/article/economy-policy/finance-ministry-to-clarify-on-equalisation-levy-116030700003_1.html

Difficult

Page 54: Detailed Solutions CSE-PRELIMS 8 • PAPER-1 · 2018-06-07 · 2 | Detailed of Solutions of CSE-Prelims:2018 • Paper-1 Analysis of 2018 Prelims: 1. The question paper was difficult

54 | Detailed of Solutions of CSE-Prelims:2018 • Paper-1

94

Consider the following statements:

1. The Fiscal Responsibility and BudgetManagement (FRBM) Review Committee Reporthasrecommended a debt to GDP ratio of 60% for thegeneral (combined) government by 2023,comprising 40% for the Central Government and20% for the State Governments.2. The Central Government has domestic liabilitiesof 21% of GDP as compared to that of 49% ofGDP of the State Governments.3. As per the Constitution of India, it is mandatoryfor a State to take the Central overnment’sconsent for raising any loan if the former owes anyoutstanding liabilities to the latter.

Which of the statements given above is/arecorrect?(a) 1 only(b) 2 and 3 only(c) 1 and 3 only(d) 1, 2 and 3

cEconomic and Social

Development

Current Applied

Fiscal Policy

Statement 1 is correct: The FiscalResponsibility and BudgetManagement (FRBM) ReviewCommittee report, has preferred adebt to GDP ratio of 60% for thegeneral government by 2023,comprising 40% for the Centralgovernment and 20% for the Stategovernments.Statement 2 is not correct: TheUnion government, which has largerdomestic liabilities of 49.23% of GDPas compared to that of the States(21% of GDP).

Statement 3 is correct: UnderClause (3) of Article 293 ofConstitution of India, it is mandatoryfor a State to take the Centralgovernment’s consent for raising anyloan if the former owes anyoutstanding liabilities to the latter.

https://dea.gov.in/sites/default/files/Volume%201%20FRBM%20Review%20Committee%20Report.pdf

Medium

95

Consider the following statements:1. The quantity of imported edible oils is more thanthe domestic production of edible oils in the lastfive years.2. The Government does not impose any customsduty on all the imported edible oils as a specialcase.

Which of the statements given above is/arecorrect?(a) 1 only(b) 2 only(c) Both 1 and 2(d) Neither 1 nor 2

aEconomic and Social

Development

Current Applied

Agriculture

Statement 1 is correct: In FY2016India’s total edible oil demand stoodat 24 mn tonnes out of which 9 mntonnes was met from domesticproduction and 15 mn tonnes metfrom imports. The latter valued ataround Rs. 65,000 crore, constitutedaround 2.5% of India’s total importbill. Statement 2 is not correct:Government can impose anycustoms or import duty on importededible seeds to support its ownfarmers.

https://economictimes.indiatimes.com/news/economy/agriculture/india-still-highly-dependent-on-edible-oil-imports-icra/articleshow/60360710.cms

https://www.reuters.com/article/india-vegoils/india-raises-import-tax-on-edible-oils-to-highest-in-a-decade-idINKBN1DI05O

Difficult

Page 55: Detailed Solutions CSE-PRELIMS 8 • PAPER-1 · 2018-06-07 · 2 | Detailed of Solutions of CSE-Prelims:2018 • Paper-1 Analysis of 2018 Prelims: 1. The question paper was difficult

Detailed of Solutions of CSE-Prelims:2018 • Paper-1 | 55

96

He wrote biographies of Mazzini, Garibaldi, Shivajiand Shrikrishna; stayed in America for some time;and was also elected to the Central Assembly. Hewas

(a) Aurobindo Ghosh(b) Bipin Chandra Pal(c) Lala Lajpat Rai(d) Motilal Nehru

cModern Indian History

Traditional Factual

Personalities of Freedo

m Struggle

Lala Lajpat Rai : Gifted with aperceptive mind, he was a prolificwriter and authored several workslike – “Unhappy India”, “Young India:An Interpretation”, “History of AryaSamaj”, “England’s Debt to India”and a series of popular biographieson Mazzini, Garibaldi and SwamiDayanand.

http://pib.nic.in/newsite/mbErel.aspx?relid=148720 Difficult

97

Consider the following statements:1. Aadhaar card can be used as a proof ofcitizenship or domicile.2. Once issued, Aadhaar number cannot bedeactivated or omitted by the Issuing Authority.

Which of the statements given above is/arecorrect?(a) 1 only(b) 2 only(c) Both 1 and 2(d) Neither 1 nor 2

d Polity and Governance

Current Applied

Inclusive

Development

Statement 1 is not correct:Aadhaar card is not a proof of Indiancitizenship, the Calcutta High Courthas ruled, while rejecting a claim ofcitizenship on the ground that theaccused possessed Aadhaar card.Statement 2 is not correct: UIDAI’supdate policy version 2.3 (availableon the official website uidai.gov.in)has provisions for deactivation ofAadhaar, which are given below:a. If within 2 years of attaining age 5,the child's biometrics are notupdated in Aadhaar database,his/her Aadhaar number will bedeactivated (no authenticationpermissible). It will be reactivatedonce biometrics are updated indatabase.b. If within 2 years of attaining age15, the child's biometrics are notupdated in Aadhaar database,his/her Aadhaar number will bedeactivated (no authenticationpermissible). It will be reactivatedonce biometrics are updated indatabase.c. If Resident has not biometricallyauthenticated in 5 years, his/her OneTime Password (OTP) basedauthentication services will bedeactivated. They will be re-activatedonce the resident biometricallyauthenticates.

https://uidai.gov.in/component/fsf/?view=faq&catid=-2&start=160

Medium

Page 56: Detailed Solutions CSE-PRELIMS 8 • PAPER-1 · 2018-06-07 · 2 | Detailed of Solutions of CSE-Prelims:2018 • Paper-1 Analysis of 2018 Prelims: 1. The question paper was difficult

56 | Detailed of Solutions of CSE-Prelims:2018 • Paper-1

98

Which of the following has/have shrunkimmensely/dried up in the recent past due tohuman activities ?

1. Aral Sea2. Black Sea3. Lake Baikal

Select the, correct answer using the code givenbelow :(a) 1 only(b) 2 and 3 only(c) 2 only(d) 1 and 3 only

d Geography Current Applied

Environmental Degrad

ation

Aral sea and Lake Baikal haveshrunk immensely and dried up inthe recent past.

https://earthobservatory.nasa.gov/Features/WorldOfChange/aral_sea.php?src=fb

https://www.nationalgeographic.com/magazine/2018/03/drying-lakes-climate-change-global-warming-drought/

Difficult

99

“Rule of Law Index” is released bywhich of the following ?(a) Amnesty International(b) International Court of Justice(c) The Office of UN Commissionerfor Human Rights(d) World Justice Project

d International Relations Pure Current

International

Institutions/Partnership

s

World Justice Project: The WJP"Rule of Law Index" measures rule oflaw adherence in 113 countries andjurisdictions worldwide.

https://worldjusticeproject.org/our-work/wjp-rule-law-index/wjp-rule-law-index-2017%E2%80%932018

Difficult

Page 57: Detailed Solutions CSE-PRELIMS 8 • PAPER-1 · 2018-06-07 · 2 | Detailed of Solutions of CSE-Prelims:2018 • Paper-1 Analysis of 2018 Prelims: 1. The question paper was difficult

Detailed of Solutions of CSE-Prelims:2018 • Paper-1 | 57

100

Which one of the following links all the ATMs inIndia ?

(a) Indian Banks’ Association(b) National Securities Depository Limited(c) National Payments Corporationof India(d) Reserve Bank of India

cEconomic and Social

Development

Traditional Factual Banking

National Financial switch under theNPCI links all ATMs in India.National Financial Switch (NFS) ATMnetwork having 37 members andconnecting about 50,000 ATMs wastaken over by NPCI from Institute forDevelopment and Research inBanking Technology (IDRBT) onDecember 14, 2009. Over the spanof few years, NFS ATM network hasgrown many folds and is now theleading multilateral ATM network inthe country. As on 31 st August’ 17,there were 941 members thatincludes 101 Direct, 776 Submembers, 56 RRBs and 8 WLAOsusing NFS network connected tomore than 2.37 Lac ATM.NFS has established a strong andsustainable operational model with in-house capabilities and today can becompared at par with other majorand well-established switchnetworks. The operational functionsand services are at par with most ofthe global ATM networks.

https://www.npci.org.in/product-overview/national-financial-switch-product-overview

Medium